67
1 Prelim IAS Test Series (2019) GS Test 8 (20.01.2019) Polity 2 and Current Affairs (November 2018) Answer Key & Exp for GS Prelim Test-8 held on 20 th Jan’ 2019 1. Consider the following statements: (1) The Fundamental Duties enshrined in the Indian Constitution are applicable to the citizens of India as well as foreigners. (2) The Parliament of India can enforce the Fundamental Duties through suitable legislation. Which of the statements given above is/are correct? (a) 1 only (b) 2 only (c) Both 1 and 2 (d) Neither 1 nor 2 Answer: (b) Explanation: The Fundamental Duties (Chapter IV-A, Article 51-A) are intended to serve as a constant reminder to every Indian citizen that while the Constitution specifically conferred on them certain Fundamental Rights, it also requires citizens of India to observe certain basic norms of democratic conduct and democratic behaviour because rights and duties are co-relative. Statement 1 is not correct: The Fundamental Duties are confined to the citizens of India only and do not extend to the foreigners unlike some of the Fundamental Rights which extend to all persons whether citizens or foreigners. Statement 2 is correct: The Fundamental Duties are also non-justiciable like the Directive Principles of State Policy. The Constitution does not provide for their direct enforcement by the courts. Moreover, there is no legal sanction against their violation. However, the Parliament of India is free to enforce them by suitable legislation. Sources: Vajiram & Ravi Institute Yellow Book; Part 1, Ch.10: Fundamental Duties, Pg.100-102 Indian Polity by M Laxmikanth- Chapter 9: Fundamental Duties, page no. 9.2 2. The Comptroller and Auditor General of India (CAG) is a constitutional body mandated to keep a watch on the system of governance in the country, to ensure the best use of taxpayers’ money. In this context, consider the following statements: (1) The salary and conditions of service of the CAG are mentioned in the Indian Constitution. (2) The CAG can be removed by the President on the same ground as that of a Supreme Court judge. (3) The CAG can be appointed as a Vice-Chancellor of the State Universities after retirement. Which of the statements given above is/are correct? (a) 3 only (b) 1 and 3 only (c) 2 and 3 only

Answer Key & Exp for GS Prelim Test-8 held on 20 …...1 Polity 2 Prelim IAS Test Series (2019) –GS Test 8 (20.01.2019) and Current Affairs (November 2018) Answer Key & Exp for GS

  • Upload
    others

  • View
    28

  • Download
    1

Embed Size (px)

Citation preview

Page 1: Answer Key & Exp for GS Prelim Test-8 held on 20 …...1 Polity 2 Prelim IAS Test Series (2019) –GS Test 8 (20.01.2019) and Current Affairs (November 2018) Answer Key & Exp for GS

1

Prelim IAS Test Series (2019) – GS Test 8 (20.01.2019) Polity 2 and Current Affairs (November 2018)

Answer Key & Exp for GS Prelim Test-8 held on 20th Jan’ 2019

1. Consider the following statements: (1) The Fundamental Duties enshrined in the Indian Constitution are applicable to

the citizens of India as well as foreigners. (2) The Parliament of India can enforce the Fundamental Duties through suitable

legislation. Which of the statements given above is/are correct? (a) 1 only (b) 2 only (c) Both 1 and 2 (d) Neither 1 nor 2

Answer: (b) Explanation: The Fundamental Duties (Chapter IV-A, Article 51-A) are intended to serve as a constant reminder to every Indian citizen that while the Constitution specifically conferred on them certain Fundamental Rights, it also requires citizens of India to observe certain basic norms of democratic conduct and democratic behaviour because rights and duties are co-relative. Statement 1 is not correct: The Fundamental Duties are confined to the citizens of India only and do not extend to the foreigners unlike some of the Fundamental Rights which extend to all persons whether citizens or foreigners. Statement 2 is correct: The Fundamental Duties are also non-justiciable like the Directive Principles of State Policy. The Constitution does not provide for their direct enforcement by the courts. Moreover, there is no legal sanction against their violation. However, the Parliament of India is free to enforce them by suitable legislation. Sources:

Vajiram & Ravi Institute Yellow Book; Part 1, Ch.10: Fundamental Duties, Pg.100-102

Indian Polity by M Laxmikanth- Chapter 9: Fundamental Duties, page no. 9.2 2. The Comptroller and Auditor General of India (CAG) is a constitutional body

mandated to keep a watch on the system of governance in the country, to ensure the best use of taxpayers’ money. In this context, consider the following statements: (1) The salary and conditions of service of the CAG are mentioned in the Indian

Constitution. (2) The CAG can be removed by the President on the same ground as that of a

Supreme Court judge. (3) The CAG can be appointed as a Vice-Chancellor of the State Universities

after retirement. Which of the statements given above is/are correct? (a) 3 only (b) 1 and 3 only (c) 2 and 3 only

Page 2: Answer Key & Exp for GS Prelim Test-8 held on 20 …...1 Polity 2 Prelim IAS Test Series (2019) –GS Test 8 (20.01.2019) and Current Affairs (November 2018) Answer Key & Exp for GS

2

Prelim IAS Test Series (2019) – GS Test 8 (20.01.2019) Polity 2 and Current Affairs (November 2018)

(d) None of the above

Answer: (d) Explanation: Statement 1 is incorrect: The salary and conditions of service of the CAG is laid down by Indian Parliament by law and these are not liable to variation to his disadvantage during his term in office. The Parliament has enacted the Comptroller and Auditor-General's (Conditions of Service) Act, 1971. Statement 2 is correct: The CAG is appointed by the President and holds office for 6 years or till the age of 65 whichever is earlier. He can be removed by the President of India on same grounds and manner as a Supreme Court Judge. He can also resign by writing to the President. Statement 3 is incorrect: After retirement, the CAG is not eligible for any office under the Government of India or a State Government. Hence, the CAG cannot be appointed as a Vice-Chancellor of the State Universities. Sources:

Indian Polity by M. Laxmikanth- Ch-49, CAG Page number 49.1, 49.2, 49.3 3. Consider the following statements regarding composition and functions of the

Finance Commission of India: (1) It is a quasi-judicial body. (2) It consists of a Chairman and four other members to be appointed by the

Prime Minister. (3) Its members are eligible for reappointment. (4) The Constitution has specified the qualifications of its Chairman and

members. Which of the statements given above is/are correct? (a) 1 only (b) 1 and 2 only (c) 1 and 3 only (d) 1, 2, 3 and 4

Answer: (c) Explanation: Finance Commission of India: Article 280 of the Constitution of India provides for a Finance Commission as a quasi-judicial body. It is constituted by the President of India every fifth year or at such earlier time as he considers necessary. Hence, Statement 1 is correct. The Finance Commission consists of a Chairman and four other members to be appointed by the President. They hold office for such period as specified by the President in his order. Hence, Statement 2 is not correct. The members of the Finance Commission are eligible for reappointment. Hence, Statement 3 is correct. The Constitution authorises the Parliament to determine the qualifications of members of the Commission and the manner in which they should be selected. Accordingly, the

Page 3: Answer Key & Exp for GS Prelim Test-8 held on 20 …...1 Polity 2 Prelim IAS Test Series (2019) –GS Test 8 (20.01.2019) and Current Affairs (November 2018) Answer Key & Exp for GS

3

Prelim IAS Test Series (2019) – GS Test 8 (20.01.2019) Polity 2 and Current Affairs (November 2018)

Parliament has specified the qualifications of the Chairman and members of the Commission. Hence, Statement 4 is not correct. Additional Information: The Chairman of the Finance Commission should be a person having experience in public affairs and the four other members should be selected from amongst the following: i. A judge of the High Court or one qualified to be appointed as one. ii. A person who has specialised knowledge of finance and accounts of the government. iii. A person who has wide experience in financial matters and in administration. iv. A person who has special knowledge of economics. Sources:

Vajiram & Ravi Yellow Book; Part 2, Ch. 7: Finance Commission, Pg.87

Indian Polity by M. Laxmikanth, Ch. 45: Finance Commission, page 45.1 4. When the Chief Justice of a High Court in India acts in his administrative capacity,

he is subjected to which of the following? (a) The writ jurisdiction of any other High Court. (b) Special control exercised by the Chief Justice of India. (c) Discretionary powers of the President of India. (d) The writ jurisdiction of the High Court of which he is a Judge.

Answer: (a) Explanation: Article 226 of the Indian Constitution empowers the High Courts to issue writs. Also, when the Chief Justice of a High Court acts in his administrative capacity, he is subject to the writ jurisdiction of the other High Courts. When the Chief Justice of a High Court in India acts in his administrative capacity, he is not subject to:

Special control exercised by the Chief Justice of India.

Discretionary powers of the President of India.

The writ jurisdiction of the High Court of which he is a Judge. Hence, option (a) is the correct answer. Sources:

Vajiram & Ravi Yellow Book; Part 2, Ch.2: The Judiciary, Pg.34-37

Indian Polity by M. Laxmikanth, Ch. 27, Judicial Review, Pg. 27.1 5. With reference to the term of a Governor’s office in an Indian State, which of the

following statements is/are correct? (1) The Governor has no security of tenure and no fixed term of office. (2) The Constitution does not lay down any grounds upon which a Governor may

be removed by the President. (3) The Supreme Court has held that the pleasure of the President in removal of

the Governor is subject to judicial review. Select the correct answer using the code given below: (a) 1 only (b) 3 only

Page 4: Answer Key & Exp for GS Prelim Test-8 held on 20 …...1 Polity 2 Prelim IAS Test Series (2019) –GS Test 8 (20.01.2019) and Current Affairs (November 2018) Answer Key & Exp for GS

4

Prelim IAS Test Series (2019) – GS Test 8 (20.01.2019) Polity 2 and Current Affairs (November 2018)

(c) 1 and 2 only (d) 1, 2 and 3

Answer: (d) Explanation: The normal term of a Governor's office is five years from the date on which he enters upon his office. However, this term of five years is subject to the ‘pleasure of the President of India’. Statement 1 is correct: The Governor has no security of tenure and no fixed term of office. He may be removed by the President at any time. Statement 2 is correct: The Constitution does not lay down any grounds upon which a Governor may be removed by the President. Statement 3 is correct: The Supreme Court held that the pleasure of the President is subject to judicial review, in B.P. Singhal vs. Union of India 2010 case. Further, he can resign at any time by addressing a resignation letter to the President. Hence, the correct answer is option (d). Sources:

Vajiram & Ravi Yellow Book; Part 2, Ch.1: Government of the State, Pg.1-3

Indian Polity 5th Edition M. Laxmikanth; Chapter 30, Pg 30.3-30.4 6. Consider the following statements about the implementation of Lokpal and

Lokayukta in India: (1) More than half of the States in India have State Lokayukta Acts in place. (2) The Lokpal will have the power of superintendence and direction over any

investigation agency including the Central Bureau of Investigation (CBI), over cases that are referred to them by the Lokpal.

(3) The President of India is the sole authority to select and appoint the Lokpal. Which of the statements given above is/are correct? (a) 3 only (b) 1 and 2 only (c) 1 and 3 only (d) 2 and 3 only

Answer: (b) Explanation: Statement 1 is correct: By 2013, 18 States and 1 Union Territory (Delhi) had established the institution of Lokayukta. Thus, more than half of the States in India have State Lokayuktas. Statement 2 is correct: The Lokpal will have the power of superintendence and direction over any investigation agency including the Central Bureau of Investigation, for cases referred to them by the Lokpal. Statement 3 is not correct: Selection of Chairperson and members of Lokpal through a selection committee consisting of the Prime Minister, the Speaker of Lok Sabha, the Leader of Opposition in Lok Sabha, the Chief Justice of India (CJI) or a sitting Supreme Court Judge nominated by the CJI.

Page 5: Answer Key & Exp for GS Prelim Test-8 held on 20 …...1 Polity 2 Prelim IAS Test Series (2019) –GS Test 8 (20.01.2019) and Current Affairs (November 2018) Answer Key & Exp for GS

5

Prelim IAS Test Series (2019) – GS Test 8 (20.01.2019) Polity 2 and Current Affairs (November 2018)

Sources:

https://www.thehindu.com/news/national/all-you-need-to-know-about-the-lokpal-Bill/Article18254568.ece

http://www.prsindia.org/administrator/uploads/media/Lok%20Pal%20Bill%202011/Amendments%20to%20Lokpal%20Bill.pdf

7. Consider the following statements regarding the Election Commission of India:

(1) The Constitution of India provides for a three-member Election Commission. (2) The Chief Election Commissioner holds his office during the pleasure of the

President. (3) The Election Commission of India administers elections to the Lok Sabha, the

Rajya Sabha and the office of the President only. Which of the statements given above is/are not correct? (a) 1 and 2 only (b) 2 and 3 only (c) 1 and 3 only (d) 1, 2 and 3

Answer: (d) Explanation: Statement 1 is not correct. As per Article 324(2) of the Indian Constitution, the Election Commission shall consist of the Chief Election Commissioner and such number of other Election Commissioners, if any, as the President may from time to time fix and the appointment of the Chief Election Commissioner and other Election Commissioners shall, subject to the provisions of any law made in that behalf by the Parliament, be made by the President. Statement 2 is not correct. As per Article 324(5) of Indian Constitution, the Chief Election Commissioner shall not be removed from his office except in like manner and on the like grounds as a Judge of the Supreme Court and the conditions of service of the Chief Election Commissioner shall not be varied to his disadvantage after his appointment. Statement 3 is not correct. The Election Commission of India is an autonomous Constitutional authority responsible for administering election processes in India. The body administers elections to the Lok Sabha, Rajya Sabha, State Legislative Assemblies in India, and the offices of the President and the Vice President in India. Source: Vajiram & Ravi yellow book, Part 2; Ch. 10: Election; Page No.127 8. Which of the following is the constitutional authority in India, vested with the power

of declaring certain castes and tribes as the Scheduled Castes (SCs) and Scheduled Tribes (STs) in a State? (a) The Prime Minister of India (b) The Governor of a State concerned (c) The President of India (d) The Chairman of the National Commission For Scheduled Castes / National

Commission For Scheduled Tribes

Page 6: Answer Key & Exp for GS Prelim Test-8 held on 20 …...1 Polity 2 Prelim IAS Test Series (2019) –GS Test 8 (20.01.2019) and Current Affairs (November 2018) Answer Key & Exp for GS

6

Prelim IAS Test Series (2019) – GS Test 8 (20.01.2019) Polity 2 and Current Affairs (November 2018)

Answer: (c) Explanation: The Constitution of India does not specify the castes or tribes which are to be called the Scheduled Castes or the Scheduled Tribes. It leaves to the President of India, the power to specify as to which castes or tribes in each State and Union Territory are to be treated as the SCs and STs. Sources:

Vajiram & Ravi Institute yellow book, Part 2; Ch.9: Miscellaneous Topics; Pg.94-97

Polity by Laxmikanth Chapter 63: Special Provisions Relating to Certain Classes-Pg. 63.1

9. With reference to the representation for Anglo-Indians in the State Legislatures,

consider the following statements: (1) The Governor of a State can nominate one member of the Anglo-Indian

community to the State Legislative Assembly. (2) Under the 95th Constitutional Amendment Act 2009; the special representation

is to last until 2020. Select the correct answer using the code given below: (a) 1 only (b) 2 only (c) Both 1 and 2 (d) Neither 1 nor 2

Answer: (c) Explanation: Statement (1) is correct: The Governor of a State can nominate one member of the Anglo-Indian community to the State Legislative Assembly, if the community is not adequately represented. Statement (2) is correct. Originally, this provision of special representation was to operate up to 1960 only. But this duration has been extended continuously since then by ten years each time. Now, under the 95th Constitutional Amendment Act of 2009, this provision of special representation is to last until 2020. Source: Indian Polity by Laxmikanth Chapter 63: Special Provisions Relating to Certain Classes-Pg. 63.2 10. In the Indian context, which of the following Fundamental Rights cannot be

suspended during the period of proclamation of a National Emergency? (a) Freedoms under Article 19 of the Constitution of India. (b) Freedom to move the court for the enforcement of Fundamental Rights. (c) Protection of life and personal liberty under Article 21. (d) Freedom to manage religious affairs under Article 26.

Answer: (c)

Page 7: Answer Key & Exp for GS Prelim Test-8 held on 20 …...1 Polity 2 Prelim IAS Test Series (2019) –GS Test 8 (20.01.2019) and Current Affairs (November 2018) Answer Key & Exp for GS

7

Prelim IAS Test Series (2019) – GS Test 8 (20.01.2019) Polity 2 and Current Affairs (November 2018)

Explanation: Articles 358 and 359 of the Constitution of India describe the effect of a National Emergency on the Fundamental Rights. According to Article 358, when a proclamation of National Emergency is made, the six Fundamental Rights under Article 19 are automatically suspended. No separate order for their suspension is required. Article 359 authorises the President to suspend the right to move any court for the enforcement of Fundamental Rights during a National Emergency. This means that under Article 359, the Fundamental Rights as such are not suspended, but only their enforcement. However, the 44th Amendment Act of 1978 restricted the scope of Article 359 in two ways. Firstly, the President cannot suspend the right to move the Court for the enforcement of Fundamental Rights guaranteed by Article 20 (Right to protection in respect of conviction for offences) and Article 21 (Right to life and personal liberty). Hence, option (c) is the correct answer. Sources:

Vajiram & Ravi Yellow Book, Part 1; Ch.8: Fundamental Rights; Page No.86-87

Indian Polity by M Laxmikanth- Ch. 7: Fundamental Rights, 16, page no. 16.4 11. Which of the following Committees/Commissions are associated with the

Fundamental Duties enumerated in the Constitution of India? (1) Sarkaria Commission (2) Justice J S Verma Committee (3) Mani Shankar Aiyar Committee (4) Sardar Swaran Singh Committee Select the correct answer using the code given below: (a) 2 and 4 only (b) 1, 2 and 4 only (c) 2, 3 and 4 only (d) 1, 2, 3 and 4

Answer: (a) Explanation: Sardar Swaran Singh Committee: In 1976, the Congress Party set up the Sardar Swaran Singh Committee to make recommendations about Fundamental Duties, the need and necessity of which was felt during the operation of the internal emergency (1975–1977). The Committee recommended the inclusion of a separate chapter on Fundamental Duties in the Constitution. Hence, point (4) is correct. Justice J. S. Verma Committee on Fundamental Duties of the citizens (1999): It identified the existence of legal provisions for the implementation of some of the Fundamental Duties. Prevention of Insults to the National Honour Act (1971) Protection of Civil Rights Act (1955) Indian Penal Code (IPC) Unlawful Activities (Prevention) Act (1967)

Page 8: Answer Key & Exp for GS Prelim Test-8 held on 20 …...1 Polity 2 Prelim IAS Test Series (2019) –GS Test 8 (20.01.2019) and Current Affairs (November 2018) Answer Key & Exp for GS

8

Prelim IAS Test Series (2019) – GS Test 8 (20.01.2019) Polity 2 and Current Affairs (November 2018)

The Representation of the People Act (1951) provides for the disqualification of members of the Parliament or a State Legislature for indulging in corrupt practice.

The Wildlife (Protection) Act (1972) The Forest (Conservation) Act of 1980 The various criminal laws in force provide for punishments for encouraging enmity

between different sections of people on grounds of language, race, place of birth, religion and so on.

Hence, point (2) is correct. Sarkaria Commission: It was set up in 1983 by the central government of India to examine the Centre-State relationship on various portfolios and suggest changes within the framework of the Constitution of India. Hence, point (1) is not correct. Mani Shankar Ayer Committee: He chaired the Coordination Committee of Zonal Cultural Centres (CCZCC). Hence, point (3) is not correct. Hence, option (c) is the correct answer. Sources:

Vajiram & Ravi yellow book, Part 1; Ch.10: Fundamental Duties; Page No.100

Indian Polity by M Laxmikanth- Chapter 9: Fundamental Duties, page no. 9.1, 9.3 12. The Attorney General of India (AG) is the Government of India’s Chief Legal

Advisor. In this context, consider the following statements: (1) He must be a person qualified to be appointed as a Judge of the Supreme

Court. (2) He can be made a member of any Parliamentary Committee. (3) He enjoys all the powers and privileges of a Cabinet Minister. Which of the statements given above are correct? (a) 1 and 2 only (b) 2 and 3 only (c) 1 and 3 only (d) 1, 2 and 3

Answer: (a) Explanation: Statement 1 is correct: Attorney General of India (AG) must be a person qualified to be appointed as a Judge of the Supreme Court or an eminent jurist, in the opinion of the President and must be a citizen of India. Statement 2 is correct: He has the right of audience in all courts within the territory of India. He has also the right to speak and take part in proceedings of both the Houses of the Parliament including joint sittings. But he cannot vote in the Parliament. He can also be made a member of any Parliamentary committee but in the committee also, he has no power to vote. Statement 3 is not correct: He has all the powers and privileges of a Member of the Parliament and not that of the Cabinet Minister, but without the right to vote. Sources:

Vajiram & Ravi yellow book, Part 1; Ch.18: The Parliament & Attorney General, Pg. 184

Page 9: Answer Key & Exp for GS Prelim Test-8 held on 20 …...1 Polity 2 Prelim IAS Test Series (2019) –GS Test 8 (20.01.2019) and Current Affairs (November 2018) Answer Key & Exp for GS

9

Prelim IAS Test Series (2019) – GS Test 8 (20.01.2019) Polity 2 and Current Affairs (November 2018)

Indian Polity by M. Laxmikanth, 5th edition- Ch-50, AG; Page number 50.1, 50.2 13. Consider the following statements regarding the National Commission for

Scheduled Castes (NCSCs) in India: (1) It is a statutory body and its members are appointed by the President. (2) It has all the powers of a civil court trying a suit like summoning and enforcing

the attendance of any person from any part of India and examining him on oath.

(3) It is also required to discharge similar functions with regard to the Other Backward Classes (OBCs) and the Anglo-Indian Community as it does with respect to the SCs.

Which of the statements given above is/are not correct? (a) 1 only (b) 3 only (c) 1 and 3 only (d) 2 and 3 only

Answer: (a) Explanation: Statement 1 is not correct: The National Commission for Scheduled Castes (NCSCs) is a constitutional body in the sense that it is directly established by Article 338 of the Constitution. Statement 2 is correct: The Commission, while investigating any matter or inquiring into any complaint, has all the powers of a civil court trying a suit and in particular in respect of the following matters: Summoning and enforcing the attendance of any person from any part of India and

examining him on oath; Requiring the discovery and production of any document; Receiving evidence on affidavits; Requisitioning any public record from any court or office; Issuing summons for the examination of witnesses and documents; and Any other matter which the President may determine. Statement 3 is correct: The Commission is also required to discharge similar functions with regard to the Other Backward Classes (OBCs) and the Anglo-Indian Community as it does with respect to the SCs. In other words, the Commission has to investigate all matters relating to the Constitutional and other legal safeguards for the OBCs and the Anglo-Indian Community and report to the President upon their working. Source: Vajiram & Ravi Institute yellow book, Part 2; Ch.9: Miscellaneous Topics; Pg.94-95 14. In the context of ‘functioning of the integrated judiciary in India’, which of the

following has/have been assigned the power of judicial review? (1) The Supreme Court of India (2) The High Courts (3) Lower Courts/Sub-ordinate courts

Page 10: Answer Key & Exp for GS Prelim Test-8 held on 20 …...1 Polity 2 Prelim IAS Test Series (2019) –GS Test 8 (20.01.2019) and Current Affairs (November 2018) Answer Key & Exp for GS

10

Prelim IAS Test Series (2019) – GS Test 8 (20.01.2019) Polity 2 and Current Affairs (November 2018)

(4) Lok Adalats and Nyaya Panchayats Select the correct answer using the code given below: (a) 1 only (b) 1 and 2 only (c) 1, 2 and 3 only (d) 1, 2, 3 and 4

Answer: (b) Explanation: Judicial review is a special power of the Supreme Court and the High Courts in India to scrutinize whether a law passed by the legislature or an action taken by the executive is in accordance with the provisions of the Constitution or not. If it is found that such a law or an action not in accordance with the Constitution, then Courts/Judiciary can declare them as invalid or unconstitutional. This means that the judiciary has to act as the guardian or custodian of the Constitution. The Supreme Court and the High Courts can exercise this power on two conditions: i. whether the law or the executive action which has come under scrutiny falls within the

competence of the authority that has framed it, and ii. whether it is consistent with Part III of the Constitution dealing with the Fundamental

Rights. Judicial review is an important power of the Supreme Court (under Articles 13 and 32) and the High Courts (under Article 226 of the Constitution). They have been protecting the Constitution from irresponsible laws and arbitrary use of power by the executive, by exercising this power. Our Fundamental Rights are also preserved by this power of the judiciary. Hence, Option (b) is the correct answer. Source: Vajiram and Ravi Yellow book on Indian Polity, Part 2, Ch.2, Page.20. 15. Consider the following statements:

(1) A Joint State Public Service Commission (JSPSC) in India is a statutory and not a constitutional body.

(2) A Joint State Public Service Commission (JSPSC) can be created by an act of the Parliament on the request of the State Legislatures concerned.

Which of the statements given above is/are correct? (a) 1 only (b) 2 only (c) Both 1 and 2 (d) Neither 1 nor 2

Answer: (c) Explanation: The Constitution of India makes a provision for the establishment of a Joint State Public Service Commission (JSPSC) for two or more States.

Page 11: Answer Key & Exp for GS Prelim Test-8 held on 20 …...1 Polity 2 Prelim IAS Test Series (2019) –GS Test 8 (20.01.2019) and Current Affairs (November 2018) Answer Key & Exp for GS

11

Prelim IAS Test Series (2019) – GS Test 8 (20.01.2019) Polity 2 and Current Affairs (November 2018)

While the UPSC and the SPSC are created directly by the Constitution, a JSPSC can be created by an act of the Parliament on the request of the State legislatures concerned. Thus, a JSPSC is a statutory and not a constitutional body. The two States of Punjab and Haryana had a JSPSC for a short period, after the creation of Haryana out of Punjab in 1966. Hence, option (c) is the correct answer. Source: Indian Polity by M Laxmikanth; 5th Edition, Chapter 44 Pg- 44.4 16. In the Indian context, which of the following statements is/are correct about the

State Human Rights Commission (SHRC)? (1) The SHRC can intervene in the subject related to Human Rights violation in

List II and List III of the Seventh Schedule to the Indian Constitution. (2) Its members are appointed by the Governor on the recommendation of the

State Council of Ministers. Select the correct answer using the codes given below: (a) 1 only (b) 2 only (c) Both 1 and 2 (d) Neither 1 nor 2

Answer: (a) Explanation: Statement 1 is correct: A State Human Rights Commission can inquire into violation of human rights only in respect of subjects mentioned in the State List (List-II) and the Concurrent List (List-III) of the Seventh Schedule of the Constitution. However, if any such case is already being inquired into by the National Human Rights Commission or any other Statutory Commission, then the State Human Rights Commission does not inquire into that case. Statement 2 is not correct: The State Human Rights Commission is a multi-member body consisting of a Chairperson and two members. The Chairperson and members are appointed by the Governor on the recommendations of a committee consisting of the Chief Minister as its head, the Speaker of the Legislative Assembly, the State Home Minister and the Leader of the Opposition in the Legislative Assembly. In the case of a State having Legislative Council, the Chairman of the Council and the Leader of the Opposition in the Council would also be the members of the committee. Source: Indian Polity by M. Laxmikanth; Chapter-54 (State Human Right Commission); pg-51.1. 17. With reference to ‘the Central Vigilance Commission (CVC) in India’, which of the

following statements is/are correct? (1) The CVC is concerned with the appointment of the Director of CBI and the

Director of Enforcement Directorate. (2) The CVC submits an annual report to the Union Government. (3) The CVC acts as a ‘Designated Agency’ to receive written complaints for

disclosure on any allegation.

Page 12: Answer Key & Exp for GS Prelim Test-8 held on 20 …...1 Polity 2 Prelim IAS Test Series (2019) –GS Test 8 (20.01.2019) and Current Affairs (November 2018) Answer Key & Exp for GS

12

Prelim IAS Test Series (2019) – GS Test 8 (20.01.2019) Polity 2 and Current Affairs (November 2018)

Select the correct answer using the code given below: (a) 1 only (b) 2 and 3 only (c) 1 and 3 only (d) 1, 2 and 3

Answer: (c) Explanation: Statement 1 is correct: The CVC is concerned with the appointment of the Director of the CBI; is also empowered to recommend, after consultation with the Director of CBI, appointment of officers to the post of the level of SP and above in DSPE. It is also concerned with the appointment of the Director, Enforcement Directorate, and any appointment above the level of Deputy Director and above in the Directorate of Enforcement (Enforcement Directorate). Statement 2 is not correct: The CVC has to present annually report to the President. The President places this report before each House of Parliament. Statement 3 is correct: In 2004, the Government of India authorised the CVC as the ‘Designated Agency’ to receive written complaints for disclosure on any allegation of corruption or misuse of office and recommend appropriate action. Source: General Studies Indian Polity Part-2; Vajiram & Ravi yellow book; chapter-9 (Miscellaneous Topics); pg. no-107,108,109. 18. In India, Electoral Bonds are being pitched as an alternative to cash donations made

to political parties and bring transparency in political funding. In this context, consider the following statements: (1) Electoral Bonds were introduced in the Union Budget 2017-18. (2) There is no compulsion of KYC fulfilment before issuing the Electoral Bonds. (3) The funding through Electoral Bonds has been made available only to political

parties with minimum one per cent of the votes polled in the last general election to the House of the People or a Legislative Assembly.

Which of the statements given above is/are correct? (a) 3 only (b) 1 and 3 only (c) 2 and 3 only (d) 1, 2 and 3

Answer: (b) Explanation: The Government of India in January, 2018 notified the Scheme of Electoral Bonds to cleanse the system of political funding in the country. Electoral Bond is a bearer instrument in the nature of a Promissory Note and an interest free banking instrument. A citizen of India or a body incorporated in India will be eligible to purchase the bond. Electoral Bond (s) would be issued/purchased for any value, in multiples of Rs. 1,000, Rs. 10,000, Rs. 1,00,000, Rs. 10,00,000 and Rs. 1,00,00,000 from specified Branches of the State Bank of India (SBI).

Page 13: Answer Key & Exp for GS Prelim Test-8 held on 20 …...1 Polity 2 Prelim IAS Test Series (2019) –GS Test 8 (20.01.2019) and Current Affairs (November 2018) Answer Key & Exp for GS

13

Prelim IAS Test Series (2019) – GS Test 8 (20.01.2019) Polity 2 and Current Affairs (November 2018)

Statement 1 is correct. Electoral Bonds were introduced in Union budget 2017-18. Statement 2 is not correct. The purchaser would be allowed to buy Electoral Bond(s) only on due fulfilment of all the extant KYC norms and by making payment from a bank account. It will not carry the name of payee. Statement 3 is correct. Electoral Bonds would have a life of only 15 days during which it can be used for making donation only to the political parties registered under section 29A of the Representation of the People Act, 1951 and which secured not less than one per cent of the votes polled in the last general election to the House of the People or a Legislative Assembly. Sources:

Vajiram & Ravi Institute yellow book, Part 2; Ch. 10: Election; Page No.138

http://pib.nic.in/PressReleaseIframePage.aspx?PRID=1515123 19. Consider the following statements about the 'devices of Parliamentary proceedings'

in India: (1) A Starred Question requires a written answer from the Minister concerned. (2) The Zero Hour is not mentioned in the Rules of Procedures. Which of the statements given above is/are not correct? (a) 1 only (b) 2 only (c) Both 1 and 2 (d) Neither 1 nor 2

Answer: (a) Explanation: Statement 1 is incorrect: The first hour of every Parliamentary sitting is slotted as the Question Hour. During this time, the members ask questions and the Ministers usually give answers. The questions are of three kinds, namely, Starred, Unstarred and Short Notice.

A Starred Question (distinguished by an asterisk) requires an oral answer and supplementary questions can follow.

An Unstarred Question, on the other hand, requires a written answer and supplementary questions cannot follow.

A Short Notice question is one that is asked by giving a notice of less than ten days. It is answered orally.

Statement 2 is correct: The Zero Hour is not mentioned in the Rules of Procedure. Thus, it is an informal device available to the members of the Parliament to raise matters without any prior notice. The Zero Hour starts immediately after the Question Hour and lasts until the agenda for the day (i.e. regular business of the House) is taken up. It is an Indian innovation and has been in existence since 1962. Sources:

Vajiram & Ravi yellow book, Part 1; Ch.18: The Parliament & Attorney General, Pg. 160-161

Indian Polity by M. Laxmikanth, 5th edition- ch-22, Parliament, pg-22.14, 22.15

Page 14: Answer Key & Exp for GS Prelim Test-8 held on 20 …...1 Polity 2 Prelim IAS Test Series (2019) –GS Test 8 (20.01.2019) and Current Affairs (November 2018) Answer Key & Exp for GS

14

Prelim IAS Test Series (2019) – GS Test 8 (20.01.2019) Polity 2 and Current Affairs (November 2018)

20. Consider the following statements: (1) The Indian Courts can declare a law violative of any of the Directive Principles

of State Policy as unconstitutional and invalid. (2) Generally the Fundamental Rights enshrined in the Indian Constitution enjoy

supremacy over the Directive Principles of State Policy. Which of the statements given above is/are correct? (a) 1 only (b) 2 only (c) Both 1 and 2 (d) Neither 1 nor 2

Answer: (b) Explanation: Statement 1 is not correct: The Directives Principles have been criticised mainly because of their non-justiciable character. Therefore, the courts cannot declare a law violative of any of the Directive Principles of State Policy as unconstitutional and invalid. However, they can uphold the validity of a law on the ground that it was enacted to give effect to a directive. Moreover, they are like general recommendations addressed to all authorities in the Indian Union. Statement 2 is correct: Since the commencement of the Constitution, the justiciability of the Fundamental Rights and the moral obligation of the State to implement Directive Principles of State Policy were at loggerheads. In the Minerva Mills case (1980), the Supreme Court also held that the Indian Constitution is founded on the bedrock of the balance between the Fundamental Rights and the Directive Principles of State Policy. They together constitute the core of commitment to social revolution. The present position is that the Fundamental Rights enjoy supremacy over the Directive Principles of State Policy. Yet, this does not mean that the Directive Principles of State Policy cannot be implemented. The Parliament can amend the Fundamental Rights for implementing the Directive Principles of State Policy, so long as the amendment does not damage or destroy the basic structure of the Constitution. However, Directive Principles of State Policy under Article 39(b) and (c) can prevail over Fundamental Rights under Articles 14 and 19. Sources:

Vajiram & Ravi Institute yellow book, Part 1; Ch. 9: DPSP; Page No.97,98

Indian Polity by M Laxmikanth- Chapter 8: DPSP, page no. 8.4-8.6 21. With reference to the Solicitor General of India, consider the following statements:

(1) Under Article 76 of the Constitution, office and duties of the Solicitor General is duly mentioned.

(2) The Solicitor General is appointed for a period of three years by the Appointments Committee of the Union Cabinet.

Which of the statements given above is/are correct? (a) 1 only (b) 2 only (c) Both 1 and 2

Page 15: Answer Key & Exp for GS Prelim Test-8 held on 20 …...1 Polity 2 Prelim IAS Test Series (2019) –GS Test 8 (20.01.2019) and Current Affairs (November 2018) Answer Key & Exp for GS

15

Prelim IAS Test Series (2019) – GS Test 8 (20.01.2019) Polity 2 and Current Affairs (November 2018)

(d) Neither 1 nor 2 Answer: (b) Explanation: Statement 1 is not correct: The office and duties of the Attorney General of India are mentioned under Article 76 of the Constitution. While, the Solicitor General and the Additional Solicitor General’s office and duties are governed by Law Officers Rules, 1987. Statement 2 is correct: The Solicitor General is appointed for a period of three years by the Appointments Committee of the Cabinet chaired by the Prime Minister of India. Sources:

https://economictimes.indiatimes.com/news/politics-and-nation/tushar-mehta-appointed-as-the-new-solicitor-general-of-india/Articleshow/66148962.cms

Indian Polity by M. Laxmikanth, 5th edition- Ch-50, AG; Page number. 50.2 22. In the context of the National Commission for Scheduled Tribes (NCSTs) in India,

which of the following statements are correct? (1) The National Commission for Scheduled Tribes (NCSTs) is a constitutional

body. (2) It deals with measures to be taken over conferring ownership rights in respect

of all non-timber forest produce to Scheduled Tribes living in forest areas. (3) It decides measures to be taken to reduce and ultimately eliminate the

practice of shifting cultivation by tribals that lead to their continuous disempowerment and degradation of land and the environment.

Select the correct answer using the code given below: (a) 1 and 2 only (b) 2 and 3 only (c) 1 and 3 only (d) 1, 2 and 3

Answer: (d) Explanation: National Commission for STs Statement 1 is correct. Like the National Commission for Schedules Castes (SCs), the National Commission for Scheduled Tribes (STs) is also a Constitutional body in the sense that it is directly established by Article 338-A of the Constitution. In 2005, the President specified the following other functions of the Commission in relation to the protection, welfare and development and advancement of the STs: Statement 2 is correct. National Commission for STs decides on measures to be taken over conferring ownership rights in respect of minor forest produce (includes all non-timber forest produce of plant origin) to STs living in forest areas. Statement 3 is correct. National Commission for STs decides on measures to be taken to reduce and ultimately eliminate the practice of shifting cultivation by tribals that lead to their continuous disempowerment and degradation of land and the environment. Additional Provisions/Information:

Page 16: Answer Key & Exp for GS Prelim Test-8 held on 20 …...1 Polity 2 Prelim IAS Test Series (2019) –GS Test 8 (20.01.2019) and Current Affairs (November 2018) Answer Key & Exp for GS

16

Prelim IAS Test Series (2019) – GS Test 8 (20.01.2019) Polity 2 and Current Affairs (November 2018)

National Commission for STs decides on: 1. Measures to be taken to improve the efficacy of relief and rehabilitation measures for

tribal groups displaced by development projects. 2. Measures to be taken to prevent alienation of tribal people from land and to effectively

rehabilitate such people in whose case alienation has already taken place. 3. Measures to be taken to elicit maximum cooperation and involvement of tribal

communities for protecting forests and undertaking social afforestation. 4. Measures to be taken to ensure full implementation of the provisions of the

Panchayats (Extension to the Scheduled Areas) Act, 1996. 5. Measures to be taken to safeguard rights of the tribal communities over mineral

resources, water resources etc., as per law. 6. Measures to be taken for the development of tribals and to work for more viable

livelihood strategies. Sources:

Indian Polity part-2; Vajiram & Ravi yellow book; chapter-9 (Miscellaneous Topics); pg. no-96-98

Indian Polity by M. Laxmikanth 5th Edition, Ch. 47: NCST, Page No. 47.1- 47.3. 23. With reference to the Public Interest Litigation (PIL) in India, which of the following

statements is not correct? (a) A PIL can be filed to protect the Fundamental Rights of the citizens. (b) The common rule of ‘locus standi’ is applicable to the PILs. (c) PILs can be filed to direct the Government policy making in the desired

direction. (d) Two warring groups involved in a dispute in the realm of private law can’t file

PIL for settlement. Answer: (b) Explanation: Option (a) is correct: Public (or) social interest litigations is innovative strategy which has been evolved by the Supreme Court for the purpose of providing easy access to justice to the weaker sections of Indian society, who are not aware or financially not capable to seek redressal against encroachment to their Fundamental Rights. Option (b) is not correct: The important innovation in the matter of PIL is relaxation regarding ‘locus standi’, which means, not only an aggrieved party can approach a court to file a case, but also anybody else(a public spirited individual) on his/her behalf. However, the court observed that it would have to be decided from case to case as to whether person approaching the court for relief has sufficient interest and has not acted with malafide (or) political motive. Option (c) is correct: Courts normally won’t interfere in policy making domain, unless it is concerned with the areas outlined by Supreme Court under which PILs are allowed. Option (d) is correct: The dispute between two warring groups purely in the realm of private law would not be allowed to be agitated as a PIL. Hence, option (b) is the correct answer. Sources:

Page 17: Answer Key & Exp for GS Prelim Test-8 held on 20 …...1 Polity 2 Prelim IAS Test Series (2019) –GS Test 8 (20.01.2019) and Current Affairs (November 2018) Answer Key & Exp for GS

17

Prelim IAS Test Series (2019) – GS Test 8 (20.01.2019) Polity 2 and Current Affairs (November 2018)

Vajiram & Ravi Institute yellow book, Part 2; Ch. 2: The Judiciary; Page No.38,39

Indian Polity by M. Laxmikanth, Ch. 29, Public Interest Litigation, Pg. 29.3 24. With reference to the Special Officer for Linguistic Minorities in India, consider the

following statements: (1) The Constitution does not specify the qualifications, tenure, salaries and

allowances, service conditions and procedure for removal of the Special Officer for Linguistic Minorities.

(2) The Office of the Special Officer for Linguistic Minorities falls under the jurisdiction of Ministry of Minority Affairs.

Which of the statements given above is/are correct? (a) 1 only (b) 2 only (c) Both 1 and 2 (d) Neither 1 nor 2

Answer: (c) Explanation: Special Officer for Linguistic Minorities Originally, the Constitution of India did not make any provision with respect to the Special Officer for Linguistic Minorities. Later, the States Reorganisation Commission (1953-55) made a recommendation in this regard. Accordingly, the Seventh Constitutional Amendment Act of 1956 inserted a new Article 350-B in Part XVII of the Constitution. This Article contains the following provisions: 1. There should be a Special Officer for Linguistic Minorities. He is to be appointed by

the President of India. 2. It would be the duty of the Special Officer to investigate all matters relating to the

safeguards provided for linguistic minorities under the Constitution. He would report to the President upon those matters at such intervals as the President may direct. The President should place all such reports before each House of Parliament and send to the governments of the States concerned

Statement 1 is correct: It must be noted here that the Constitution does not specify the qualifications, tenure, salaries and allowances, service conditions and procedure for removal of the Special Officer for Linguistic Minorities. Statement 2 is correct: At the Central level, the Office of the Special Officer for Linguistic Minorities falls under the Ministry of Minority Affairs. Hence, he submits the annual reports or other reports to the President through the Union Minority Affairs Minister. Source: Indian Polity by M. Laxmikanth 5th Edition, Ch. 48: Special Officer for Linguistic Minorities, Page No. 48.1- 48.3. 25. The Prime Minister of India is the Chairperson of which of the following bodies?

(1) NITI Aayog (2) Inter-State Council (3) National Development Council (4) Economic Advisory Council

Page 18: Answer Key & Exp for GS Prelim Test-8 held on 20 …...1 Polity 2 Prelim IAS Test Series (2019) –GS Test 8 (20.01.2019) and Current Affairs (November 2018) Answer Key & Exp for GS

18

Prelim IAS Test Series (2019) – GS Test 8 (20.01.2019) Polity 2 and Current Affairs (November 2018)

Select the correct answer using the code given below: (a) 1, 2 and 3 only (b) 1, 3 and 4 only (c) 2, 3 and 4 only (d) 1, 2, 3 and 4

Answer: (a) Explanation: The Prime Minister is the Chairman of the Planning Commission (now NITI Aayog), National Development Council, National Integration Council, Inter-State Council and National Water Resources Council. The Economic Advisory Council (EAC) is chaired by Dr. Bibek Debroy, (member NITI Aayog). Sources:

Vajiram & Ravi yellow book; Part 1; Ch.16: Prime Minister; Pg.135-138

Indian Polity by M. Laxmikanth 5th Edition, Ch. 19: Prime Minister, Page No 19.3 26. Consider the following statements regarding different categories of Ministers at the

Union level in India: (1) The Cabinet Ministers are members of the Cabinet, attend its meetings and

play an important role in deciding policies. (2) The Ministers of State work under the supervision and guidance as well as

under the overall charge and responsibility of the Cabinet Ministers, unless given an independent charge.

(3) The Deputy Ministers are not the members of the Cabinet and do not attend the Cabinet meetings.

Which of the statements given above is/are correct? (a) 1 only (b) 1 and 2 only (c) 2 and 3 only (d) 1, 2 and 3

Answer: (d) Explanation: Composition of the Council of Ministers The Council of Ministers consists of three categories of Ministers, namely, Cabinet Ministers, Ministers of State, and Deputy Ministers. 1. The Cabinet Ministers head the important ministries of the Central Government like

Home, Defence, Finance, External affairs and so forth. They are members of the Cabinet, attend its meetings and play an important role in deciding policies. Thus, their responsibilities extend over the entire gamut of the Central Government.

2. The Ministers of State can either be given independent charge of Ministries/Departments or can be attached to Cabinet Ministers. They are not members of the Cabinet and do not attend the Cabinet meetings unless specially invited when something related to their Ministries/Departments are considered by the Cabinet.

Page 19: Answer Key & Exp for GS Prelim Test-8 held on 20 …...1 Polity 2 Prelim IAS Test Series (2019) –GS Test 8 (20.01.2019) and Current Affairs (November 2018) Answer Key & Exp for GS

19

Prelim IAS Test Series (2019) – GS Test 8 (20.01.2019) Polity 2 and Current Affairs (November 2018)

3. Deputy Ministers: Not given independent charge of the Ministries/Departments. They are attached to the Cabinet Ministers or Ministers of State and assist them in their administrative, political, and Parliamentary duties. They are not members of the Cabinet and do not attend the Cabinet meetings.

It must also be mentioned here that there is one more category of Ministers, called Parliamentary Secretaries. They are the members of the last category of the Council of Ministers (which is also known as the ‘Ministry’). They have no Department under their control. They are attached to the senior Ministers and assist them in the discharge of their Parliamentary duties. However, since 1967, no Parliamentary Secretaries have been appointed except during the first phase of Rajiv Gandhi Government. Sources:

Vajiram & Ravi Institute yellow book, Part 1; Ch. 17: Council of Ministers ; Page No.140-141

Indian Polity by M. Laxmikanth 5th Edition, Ch. 20: Central Council of Ministers, Page No. 20.4

27. With reference to the Union Territories in India, consider the following statements:

(1) There are at present seven Union Territories in India. (2) All the Union Territories have Legislative Assemblies. Which of the statements given above is/are correct? (a) 1 only (b) 2 only (c) Both 1 and 2 (d) Neither 1 nor 2

Answer: (a) Explanation: Statement 1 is correct. There are 7 Union Territories in India, viz. Andaman and Nicobar Islands, Chandigarh, Daman and Diu, Dadra and Nagar Haveli, Delhi, Puducherry and Lakshadweep. Statement 2 is incorrect. Only Delhi and Puducherry have Legislative Assembly. Additional Information: All the five UTs without legislature (Andaman and Nicobar Islands, Chandigarh, Daman and Diu, Dadra and Nagar Haveli and Lakshadweep) have the forum of Home Minister’s Advisory Committee (HMAC). Source: Indian Polity by M. Laxmikanth 5th Edition, Ch. 40: Union territories, Page No. 40.6 28. Which of the following pairs is/are correctly matched?

Page 20: Answer Key & Exp for GS Prelim Test-8 held on 20 …...1 Polity 2 Prelim IAS Test Series (2019) –GS Test 8 (20.01.2019) and Current Affairs (November 2018) Answer Key & Exp for GS

20

Prelim IAS Test Series (2019) – GS Test 8 (20.01.2019) Polity 2 and Current Affairs (November 2018)

List-I (Name of High

Court )

List-II (Union Territory in jurisdiction)

1. Bombay : Dadra and Nagar Haveli

2. Madras : Andaman and Nicobar islands

3. Kerala : Lakshadweep

4. Calcutta : Puducherry

(a) 1 only (b) 1 and 2 only (c) 1 and 3 only (d) 1, 3 and 4 only

Answer: (c) Explanation: The correct pairs are: 1. Bombay High Court : Dadra and Nagar Haveli 2. Madras High Court : Puducherry 3. Kerala High Court: Lakshadweep 4. Calcutta High Court: Andaman and Nicobar islands Hence, option (c) is the correct answer. Sources:

Vajiram & Ravi Yellow book, Part 2; Ch.2: Judiciary; Page No.29,30

Indian Polity by M. Laxmikanth 5th Edition, Ch. 34: High Court, Page No. 34.14 29. Assertion (A): In India, a Minister at the State level continues in office till he enjoys

the confidence of the Chief Minister. Reason (R): The Chief Minister of the State concerned can ask the Minister to resign or advise the Governor to dismiss him in case of difference of opinion. Select the correct answer from the code given below: (a) Both A and R are true and R is the correct explanation of A. (b) Both A and R are true but R is not a correct explanation of A. (c) A is true but R is false. (d) A is false but R is true.

Answer: (a) Explanation: The Chief Minister enjoys the following powers as Head of the State Council of Ministers: (a) The Governor appoints only those persons as Ministers who are recommended by the

Chief Minister. (b) He allocates and reshuffles the portfolios among Ministers. (c) He can ask a Minister to resign or advise the Governor to dismiss him in case of

difference of opinion. (d) He presides over the meetings of the State Council of Ministers and influences its

decisions.

Page 21: Answer Key & Exp for GS Prelim Test-8 held on 20 …...1 Polity 2 Prelim IAS Test Series (2019) –GS Test 8 (20.01.2019) and Current Affairs (November 2018) Answer Key & Exp for GS

21

Prelim IAS Test Series (2019) – GS Test 8 (20.01.2019) Polity 2 and Current Affairs (November 2018)

(e) He guides, directs, controls and coordinates the activities of all the Ministers. (f) He can bring about the collapse of the State Council of Ministers by resigning from

office. Since the Chief Minister is the head of the State Council of Ministers, his resignation or death automatically dissolves the State Council of Ministers. The resignation or death of any other Minister, on the other hand, merely creates a vacancy, which the Chief Minister may or may not like to fill.

Source: Indian Polity by M. Laxmikanth 5th Edition, Ch. 31: Chief Minister, Page No. 31.4 30. Which of the following provisions of the Indian Constitution under Directive

Principles of State Policy were added through Amendments? (1) Protection and improvement of environment and safeguarding of forests and

wildlife. (2) Right to work, to education and to public assistance in cases of

unemployment. (3) Securing opportunities for healthy development of the children. Select the correct answer using the code given below: (a) 1 only (b) 2 and 3 only (c) 1 and 3 only (d) 1, 2 and 3

Answer: (c) Explanation: Article 48A: The State shall endeavour to protect and improve the environment and to safeguard the forests and wildlife of the country. This provision has been added through 42nd Constitutional Amendment Act, 1976. Article 41: The State shall, within the limits of its economic capacity and development, make effective provision for securing the right to work, to education and to public assistance in cases of unemployment, old age, sickness and disablement, and in other cases of undeserved want. This provision existed originally in the Constitution. Article 39 (f): The State shall, in particular, direct its policy towards securing - that children are given opportunities and facilities to develop in a healthy manner and in conditions of freedom and dignity and that childhood and youth are protected against exploitation and against moral and material abandonment. This provision has been added through 42nd Constitutional Amendment Act, 1976. Hence, option(c) is the correct answer. Sources:

Vajiram & Ravi Institute yellow book, Part 1; Ch. 9:DPSP; Page No.93

Indian Polity by M Laxmikanth- Chapter 8:DPSP, page no. 8.2-8.3 31. With reference to the High Court Judges in India, consider the following statements:

(1) A person, who has held office as a permanent Judge of a High Court, can plead or act in any capacity in the Supreme Court only.

Page 22: Answer Key & Exp for GS Prelim Test-8 held on 20 …...1 Polity 2 Prelim IAS Test Series (2019) –GS Test 8 (20.01.2019) and Current Affairs (November 2018) Answer Key & Exp for GS

22

Prelim IAS Test Series (2019) – GS Test 8 (20.01.2019) Polity 2 and Current Affairs (November 2018)

(2) A person holding a judicial office in India for at least 7 years is qualified for the appointment as a Judge of the High Court.

Which of the statements given above is/are correct? (a) 1 only (b) 2 only (c) Both 1 and 2 (d) Neither 1 nor 2

Answer: (d) Explanation: Statement 1 is not correct: Article 220 prohibits a person who has held the office of a permanent Judge of a High Court to plead, from acting or pleading in any court or before any authority in India except the Supreme Court and the other High Courts. Statement 2 is not correct: Article 217(2) of the Constitution provides that person shall not be qualified for appointment as a Judge of a High Court unless he is a citizen of India, and: Has for ten years held a judicial office in the territory of India, or Has at least for ten years been an advocate of High Court or of two more such courts

in succession. Sources:

Vajiram & Ravi Yellow book, Part 2; Ch.2: Judiciary; Page No.25,26

Indian Polity by M. Laxmikant, Ch. 27, Judicial Review, Pg 27.1 32. With reference to the ‘Panchayats (Extension to Scheduled Areas) Act 1996’,

consider the following statements: (1) Planning and management of minor water bodies in the Scheduled Areas is

extended to the Panchayats. (2) Prior recommendation of Gram Sabha is mandatory for the exploitation of

minor minerals by auction. Which of the statements given above is/are correct? (a) 1 only (b) 2 only (c) Both 1 and 2 (d) Neither 1 nor 2

Answer: (c) Explanation: Statement 1 is correct: Planning and management of water bodies in the scheduled areas shall be entrusted to Panchayats at the appropriate level. This is an important feature of PESA Act. Statement 2 is correct: Another important feature of the Act is that prior recommendation of the Gram Sabha or the Panchayats at the appropriate level shall be made mandatory for grant of prospective license, mining lease or concession for the exploitation of minor minerals by auction. Source: Vajiram & Ravi Institute yellow book, Part 2; Ch. 3, page- 54, 55

Page 23: Answer Key & Exp for GS Prelim Test-8 held on 20 …...1 Polity 2 Prelim IAS Test Series (2019) –GS Test 8 (20.01.2019) and Current Affairs (November 2018) Answer Key & Exp for GS

23

Prelim IAS Test Series (2019) – GS Test 8 (20.01.2019) Polity 2 and Current Affairs (November 2018)

33. Regarding the National Commission for Backward Classes (NCBC) in India, which

of the following statements is correct? (a) It lies under the jurisdiction of Ministry of Home Affairs. (b) It has been conferred a status of a constitutional body. (c) It considers inclusions in and exclusions from the lists of communities notified

as backward for the purpose of job reservations and tenders the needful advice to the Central Government.

(d) It has been empowered to look into the grievances of persons of Other Backward Classes.

Answer: (c) Explanation: National Commission for Backward Classes (NCBC): Option (a) is not correct: National Commission for Backward Classes lies under the jurisdiction of Ministry of Social Justice and Empowerment. Option (b) is not correct: National Commission for Backward Classes (NCBC) is a statutory body under Ministry of Social Justice and Empowerment. It is proposed as a constitutional body under Article 338B as per the Constitutional (123rd Amendment) Bill, 2018. Option (c) is correct: The Commission considers inclusions in and exclusions from the lists of communities notified as backward for the purpose of job reservations and tenders the needful advice to the Central Government. Option (d) is not correct: National Commission for Backward Classes has not yet been empowered to look into the grievances of persons of Other Backward Classes. Additional Information: Report of the Commission: The NCBC presents annual report to the Central Government containing full account of its activities during previous year. The Report is placed before each House of the Parliament by Central Government, along with memorandum explaining action taken on advice tendered by Commission. The memorandum also contains reasons for non-acceptance of any such advice. Indra Sawhney & Others. vs. Union of India: The Commission was the outcome of Indra Sawhney & Ors. vs. Union of India. The Supreme Court of India in its Judgement – directed the Government of India, State Governments and Union Territory administrations to constitute a permanent body in the nature of a Commission or Tribunal for entertaining, examining and recommending upon requests for inclusion and complaints of over-inclusion and under-inclusion in the list of OBCs. The Supreme Court held that the Constitution recognised only social and educational but not economic backwardness. Sources:

Indian Polity part-2; Vajiram & Ravi yellow book; chapter-9 (Miscellaneous Topics); pg. no-104-106

Indian Polity by M. Laxmikanth 5th Edition, Ch. 47: NCBC, Page No. 47.1- 47.3 34. Which of the following correctly indicates the status of the NITI Aayog?

(a) A constitutional body

Page 24: Answer Key & Exp for GS Prelim Test-8 held on 20 …...1 Polity 2 Prelim IAS Test Series (2019) –GS Test 8 (20.01.2019) and Current Affairs (November 2018) Answer Key & Exp for GS

24

Prelim IAS Test Series (2019) – GS Test 8 (20.01.2019) Polity 2 and Current Affairs (November 2018)

(b) A statutory but extra-constitutional body (c) A non-statutory and extra-constitutional body (d) None of the above

Answer: (c) Explanation: The National Institution for Transforming India, also called NITI Aayog, was formed via a resolution of the Union Cabinet on January 1, 2015. The Government of India, in keeping with its reform agenda, constituted the NITI Aayog to replace the Planning Commission instituted in 1950. As, it has neither been established by Constitution, not by an act of Parliament, it is a Non-Statutory and extra-Constitutional body. Hence, Option (c) is the correct answer. Source: Indian Polity by M. Laxmikanth, Ch. 52, NITI Aayog, Page 52.1 35. Which of the following functions is not performed by the Union Public Service

Commission (UPSC) in India? (a) Assisting the States in framing and operating schemes of joint recruitment for

any services. (b) Consulting in all disciplinary matters affecting a person serving under the

Government of India in a civil capacity. (c) Determining the classification of services, pay and service conditions. (d) Acting as the Central Recruiting Agency.

Answer: (c) Explanation: The Constitution visualises the UPSC to be the ‘watch-dog of merit system’ in India. It is concerned with the recruitment to the all-India services and Central services -group A and group B and advises the government, when consulted, on promotion and disciplinary matters. It is not concerned with the classification of services, pay and service conditions, cadre management, training, and so on. These matters are handled by the Department of Personnel and Training—one of the three departments of the Ministry of Personnel, Public Grievances and Pensions. Therefore, UPSC is only a central recruiting agency while the Department of Personnel and Training is the central personnel agency in India. Functions of the UPSC are:

It conducts examinations for appointments to the all-India services, Central services and public services.

Assists the States in framing and operating schemes of joint recruitment for any services for which candidates possessing special qualifications are required.

It serves all or any of the needs of a State on the request of the State Governor and with the approval of the President of India.

It is consulted on matters related to personnel management. Hence, option(c) is the correct answer.

Page 25: Answer Key & Exp for GS Prelim Test-8 held on 20 …...1 Polity 2 Prelim IAS Test Series (2019) –GS Test 8 (20.01.2019) and Current Affairs (November 2018) Answer Key & Exp for GS

25

Prelim IAS Test Series (2019) – GS Test 8 (20.01.2019) Polity 2 and Current Affairs (November 2018)

Source: Indian Polity by M. Laxmikanth, 5th Edition, Chapter 43 Pg-43.2 36. River Water Disputes are a recurring subject in Indian polity. In this context,

consider the following statements: (1) Under Article 262 of the Constitution the Parliament may by law provide for

the adjudication of any dispute or complaint with respect to the use, distribution and control of waters of any inter-State river and river valley.

(2) The President may also provide that neither the Supreme Court nor any other court can exercise jurisdiction in respect of any such dispute or complaint.

(3) The Inter-State Water Disputes Act empowers the Central government to set up a tribunal which is permanent in nature.

(4) The decision of the tribunal would be final and binding on the parties to the dispute

Which of the statements given above are correct? (a) 1 and 2 only (b) 1 and 4 only (c) 1, 3 and 4 only (d) 1, 2 and 4 only

Answer: (b) Explanation: Article 262: Article 262 of the Constitution provides for the adjudication of inter-State water disputes. Statement 1 is correct: The Parliament may by law provide for the adjudication of any dispute or complaint with respect to the use, distribution and control of waters of any inter-State river and river valley. Statement 2 is not correct: The Parliament may also provide that neither the Supreme Court nor any other court is to exercise jurisdiction in respect of any such dispute or complaint. Statement 3 is not correct: The Inter-State Water Disputes Act empowers the Central government to set up an ad hoc tribunal for the adjudication of a dispute between two or more States in relation to the waters of an inter-State river or river valley. Statement 4 is correct: The decision of the tribunal would be final and binding on the parties to the dispute. Neither the Supreme Court nor any other court is to have jurisdiction in respect of any water dispute which may be referred to such a tribunal under this Act. Sources:

M. Laxmikanth: Indian Polity; Ch. 15: Inter-State Relations; Page 15.1

https://www.thehindu.com/news/national/after-cauvery-a-look-at-other-inter-State-water-tussles/Article22813510.ece

37. In the context of ‘the Central Bureau of Investigation (CBI) in India’, which of the

following statements is correct? (a) The CBI in its present form was created in 1964 by enactment of Delhi Special

Police Establishment Act.

Page 26: Answer Key & Exp for GS Prelim Test-8 held on 20 …...1 Polity 2 Prelim IAS Test Series (2019) –GS Test 8 (20.01.2019) and Current Affairs (November 2018) Answer Key & Exp for GS

26

Prelim IAS Test Series (2019) – GS Test 8 (20.01.2019) Polity 2 and Current Affairs (November 2018)

(b) The CBI possesses functional and financial autonomy granted by the statute. (c) The CBI functions under the Cabinet Secretariat. (d) The CBI provides assistance to the Central Vigilance Commission and the

Lokpal. Answer: (d) Explanation: Option (a) is not correct: The CBI was created in 1963 by a resolution of the Ministry of Home Affairs. It’s establishment was recommended by K. Santhanam committee on Prevention of Corruption (1962-1964). Option (b) is not correct: The CBI is not a statutory body. It does not enjoy financial autonomy. Option (c) is not correct: The CBI enjoys the status of attached office of Ministry of Personnel, coming under the administrative control of Department of Personnel and Training (DoPT). Option (d) is correct: As per the Central Vigilance Commission (CVC) Act, 2003 and the Lokpal & Lokayuktas Act, 2013; the CBI is required to provide investigative assistance to CVC and Lokpal. Sources: Indian Polity part-2; Vajiram & Ravi yellow book; chapter-9 (Miscellaneous Topics); pg. no-110,111 38. With reference to ‘the differences between Constitution of the Supreme Court and

the High Courts’ in India, consider the following statements: (1) The Judges of the Supreme Court are appointed by the President of India

while the Judges of the High Court are appointed by the Governor of the State concerned.

(2) A Judge of the Supreme Court can be impeached by the Parliament while a Judge of the High Court can be impeached by the State Legislative Assembly.

(3) The territorial jurisdiction of the Supreme Court extends all over India while the territorial jurisdiction of the High Court is co-terminus with the territory of the State concerned.

Which of the statements given above is/are correct? (a) 1 only (b) 2 only (c) 3 only (d) None of the above

Answer: (c) Explanation: Statement 1 is not correct: The Judges of both the Supreme Court and the High Courts are appointed by the President of India. However, he needs to go through the prescribed consultation process with the Collegium of senior-most Judges before doing so. Statement 2 is not correct: The Judges of both the Supreme Court and the High Courts can be removed by President based on the 'Impeachment Motion' passed against the Judges concerned only by the Parliament of India.

Page 27: Answer Key & Exp for GS Prelim Test-8 held on 20 …...1 Polity 2 Prelim IAS Test Series (2019) –GS Test 8 (20.01.2019) and Current Affairs (November 2018) Answer Key & Exp for GS

27

Prelim IAS Test Series (2019) – GS Test 8 (20.01.2019) Polity 2 and Current Affairs (November 2018)

Statement 3 is correct: The territorial jurisdiction of the High Court is co-terminus with the territory of the State. However, the Parliament can create a High Court for 2 or more States, or one State and one or more Union Territories. Source: Vajiram & Ravi Institute Yellow Book, Indian Polity Part – 2, The Judiciary, Page 21-23, 27. 39. With respect to the expression ‘martial law’ mentioned in the Constitution of India,

consider the following statements: (1) It has not been defined anywhere in the Constitution. (2) It has specific and detailed provisions in the Constitution. (3) It is imposed in the whole of India. Which of the statements given above is/are correct? (a) 1 only (b) 1 and 2 only (c) 2 and 3 only (d) 1, 2 and 3

Answer: (a) Explanation: Article 34 provides for the restrictions on Fundamental Rights while martial law is in force in any area within the territory of India. It empowers the Parliament to indemnify any government servant or any other person for any act done by him in connection with the maintenance or restoration of order in any area where martial law was in force. The Parliament can also validate any sentence passed, punishment inflicted, forfeiture ordered, or other act done under martial law in such area. Statement 1 is correct: The concept of martial law has been borrowed in India from the English common law. However, the expression ‘martial law’ has not been defined anywhere in the Constitution. Literally, it means ‘military rule’. It refers to a situation where civil administration is run by the military authorities according to their own rules and regulations framed outside the ordinary law. Statement 2 is not correct: There is also no specific or express provision in the Constitution that authorises the executive to declare martial law. However, it is implicit in Article 34 under which martial law can be declared in any area within the territory of India. Statement 3 is not correct: Martial law is imposed in some specific area of the country unlike National emergency which can be imposed in the whole country or in any part of it. The declaration of a martial law under Article 34 is different from the declaration of a national emergency under Article 352. Sources:

Vajiram & Ravi yellow book, Part1; Ch.8 Fundamental Rights; Pg. 86

Indian Polity by M Laxmikanth-Ch. 7: Fundamental Rights, page no. 7.21 40. Regarding functions of a State Public Service Commission (SPSC) in India, which of

the following statements is/are correct? (1) The jurisdiction of a SPSC can be extended by an Act of the Parliament.

Page 28: Answer Key & Exp for GS Prelim Test-8 held on 20 …...1 Polity 2 Prelim IAS Test Series (2019) –GS Test 8 (20.01.2019) and Current Affairs (November 2018) Answer Key & Exp for GS

28

Prelim IAS Test Series (2019) – GS Test 8 (20.01.2019) Polity 2 and Current Affairs (November 2018)

(2) The Governor of the State concerned can exclude posts, services and matters from the purview of the SPSC.

Select the correct answer using the code below: (a) 1 only (b) 2 only (c) Both 1 and 2 (d) Neither 1 nor 2

Answer: (b) Explanation: A SPSC performs all those functions in respect of the State services as the UPSC does in relation to the Central Services. Apart from these, there are following provisions regarding the functions and limitations: Statement 1 is not correct. The additional functions (extended jurisdiction) relating to the services of the State can be conferred on the SPSC by the State Legislature concerned. It can also place the personnel system of any local authority, corporate body or public institution within the jurisdiction of the SPSC. Statement 2 is correct. The Governor of the State concerned can exclude posts, services and matters from the purview of the SPSC. The Constitution states that the Governor, in respect to the State services and posts may make regulations specifying the matters in which, it shall not be necessary for the SPSC to be consulted. But all such regulations made by the Governor shall be laid before each House of the State legislature for at least 14 days in advance. The State legislature can amend or repeal them. Hence, the correct answer is option (b). Source: Indian Polity by M. Laxmikanth, 5th Edition, Chapter 44 Pg 44.3.

41. Consider the following statements about the different types of funds of the

Government of India: (1) No money can be appropriated out of the Consolidated Fund or the

Contingency Fund without approval from the Parliament. (2) The money received from Departmental Deposits or Judicial Deposits are

added to the Consolidated Fund of India. (3) The Contingency Fund of India has been established by the Parliament and is

held by the Finance Secretary on behalf of the President. Which of the statements given above is/are correct? (a) 3 only (b) 1 and 2 only (c) 2 and 3 only (d) 1, 2 and 3

Answer: (a) Explanation: Statement 1 is not correct: No money can be appropriated (issued or drawn) out of the Consolidated fund of India except in accordance with a Parliamentary law. However, the

Page 29: Answer Key & Exp for GS Prelim Test-8 held on 20 …...1 Polity 2 Prelim IAS Test Series (2019) –GS Test 8 (20.01.2019) and Current Affairs (November 2018) Answer Key & Exp for GS

29

Prelim IAS Test Series (2019) – GS Test 8 (20.01.2019) Polity 2 and Current Affairs (November 2018)

Contingency Fund and the Public Accounts Fund are operated by the executive action. Statement 2 is not correct: The provident fund deposits, judicial deposits, savings bank deposits, departmental deposits, remittances and so on are deposited in the Public Account of India. Statement 3 is correct: The Constitution authorised the Parliament to establish a ‘Contingency Fund of India’, into which amounts determined by law are paid from time to time. Accordingly, the Parliament enacted the Contingency Fund of India Act in 1950. The fund is held by the Finance Secretary on behalf of the President. Additional Information: Following forms the Consolidated Fund of India: (a) All revenues received by the Government of India; (b) All loans raised by the Government by the issue of treasury Bills, loans or ways and

means of advances; and (c) All money received by the government in repayment of loans. Source: Indian Polity by M. Laxmikanth, 5th edition- ch-22, Parliament, pg-22.26 42. Which of the following languages in the original Indian Constitution were added by

means of amendments to the Eighth Schedule to the Constitution? (1) Nepali (2) Manipuri (3) Odia (4) Sindhi Select the correct answer using the code given below: (a) 1, 2 and 3 only (b) 1, 2 and 4 only (c) 2, 3 and 4 only (d) 1, 2, 3 and 4

Answer: (b) Explanation: Languages recognized by the Constitution of India: Originally, it had 14 languages but presently there are 22 languages. They are: Assamese, Bengali, Bodo, Dogri (Dongri), Gujarati, Hindi, Kannada, Kashmiri, Konkani, Mathili (Maithili), Malayalam, Manipuri, Marathi, Nepali, Odia, Punjabi, Sanskrit, Santhali, Sindhi, Tamil, Telugu and Urdu. Sindhi was added by the 21st Amendment Act of 1967. Konkani, Manipuri and Nepali were added by the 71st Amendment Act of 1992. Bodo, Dongri, Maithili and Santhali were added by the 92nd Amendment Act of 2003. Oriya was renamed as ‘Odia’ by the 96th Amendment Act of 2011 Hence, option (b) is the correct answer. Source: Indian Polity by M. Laxmikanth, 5th Edition, Chapter 3 Pg 3.10 43. With reference to the pardoning power of the President and the Governor in India,

which of the following statements is/are correct?

Page 30: Answer Key & Exp for GS Prelim Test-8 held on 20 …...1 Polity 2 Prelim IAS Test Series (2019) –GS Test 8 (20.01.2019) and Current Affairs (November 2018) Answer Key & Exp for GS

30

Prelim IAS Test Series (2019) – GS Test 8 (20.01.2019) Polity 2 and Current Affairs (November 2018)

(1) The President can pardon sentences inflicted by Court Martial (military courts) while the Governor cannot.

(2) Both the President and the Governor have concurrent powers in respect of suspension, remission and commutation of death sentence.

Select the correct answer using the code given below: (a) 1 only (b) 2 only (c) Both 1 and 2 (d) Neither 1 nor 2

Answer: (c) Explanation: In India, the pardoning power of the Governor differs from that of the President: Statement 1 is correct. The President can pardon sentences inflicted by court martial (military courts) while the Governor cannot. Statement 2 is correct. The President can pardon death sentence while Governor cannot. Even if a State law prescribes death sentence, the power to grant pardon lies with the President and not the Governor. However, the Governor can suspend, remit or commute a death sentence. In other words, both the Governor and the President have concurrent power in respect of suspension, remission and commutation of death sentence. Hence, option (c) is the correct answer. Source: Indian Polity by M Laxmikanth; 5th edition, Chapter 17, Page 17.12 44. The Electronic Voting Machines (EVMs) reduce the time in both casting a vote and

declaring the results as compared to the old paper ballot system. Consider the following statements with reference to EVMs: (1) India turned into a complete e-democracy in General Elections 2004 when

EVMs were used across all polling stations in the country. (2) EVMs are tamper-proof and cannot be connected to any external network

devices. Which of the statements given above is/are correct? (a) 1 only (b) 2 only (c) Both 1 and 2 (d) Neither 1 nor 2

Answers: (c) Explanation: With a view to overcome certain problems associated with the use of ballot papers and taking advantage of development of technology so that voters cast their votes correctly without any resultant ambiguity and removing the possibilities of invalid votes totally, the Commission in December, 1977 mooted the idea of EVM. Statement 1 is correct. Electronic Voting Machines ("EVM") are being used in Indian General and State Elections to implement electronic voting in part from 1999 elections

Page 31: Answer Key & Exp for GS Prelim Test-8 held on 20 …...1 Polity 2 Prelim IAS Test Series (2019) –GS Test 8 (20.01.2019) and Current Affairs (November 2018) Answer Key & Exp for GS

31

Prelim IAS Test Series (2019) – GS Test 8 (20.01.2019) Polity 2 and Current Affairs (November 2018)

and in total since 2004 elections. The EVMs reduce the time in both casting a vote and declaring the results compared to the old paper ballot system. Bogus voting and booth capturing can be greatly reduced by the use of EVMs. Illiterate people find EVMs easier than ballot paper system. Statement 2 is correct. The EVM machine is electronically protected to prevent any tampering/manipulation. The programme (software) used in these machines is burnt into a One Time Programmable (OTP) / Masked chip so that it cannot be altered or tampered with. Further these machines are not networked either by wire or by wireless to any other machine or system. Therefore, there is no possibility of its data corruption. EVMs are Standalone Machine and cannot be connected to any external network devices. Sources:

http://pib.nic.in/newsite/PrintRelease.aspx?relid=159351

Vajiram & Ravi Institute yellow book, Part 2; Ch. 10: Election; Page No.128 45. Consider the following statements about the Central Administrative Tribunal (CAT)

of India: (1) CAT exercises original jurisdiction in relation to the recruitment and all service

matters of the public servants covered by it. (2) Appeals against the orders of the CAT could be made only in the Supreme

Court. (3) CAT is bound by the procedure laid down in the Civil Procedure Code of 1908. Which of the statements given above are not correct? (a) 1 and 2 only (b) 1 and 3 only (c) 2 and 3 only (d) 1, 2 and 3

Answer: (c) Explanation: Statement (1) is correct: The Central Administrative Tribunal exercises original jurisdiction in relation to the recruitment and all service matters of public servants covered by it. Its jurisdiction extends to the All-India Services, the Central Civil Services, Civil Posts under the Centre and the Civilian employees of Defence Services. However, the members of the Defence Forces, Officers and public servants of the Supreme Court and the secretarial staff are not covered by it. Statement (2) is not correct: Originally, appeals against the orders of the CAT could be made only in the Supreme Court and not in the High Courts. However, the Supreme Court in Chandrakumar case (1997) declared this restriction on the jurisdiction of the High Courts as unconstitutional, holding that judicial review is a part of the basic structure of the Constitution. It laid down that appeals against the orders of the CAT shall lie before the division bench of the High Court concerned. Consequently, now it is not possible for an aggrieved public servant to approach the Supreme Court directly against an order of CAT, without first going to the High Court concerned.

Page 32: Answer Key & Exp for GS Prelim Test-8 held on 20 …...1 Polity 2 Prelim IAS Test Series (2019) –GS Test 8 (20.01.2019) and Current Affairs (November 2018) Answer Key & Exp for GS

32

Prelim IAS Test Series (2019) – GS Test 8 (20.01.2019) Polity 2 and Current Affairs (November 2018)

Statement (3) is not correct: The CAT is not bound by the procedure laid down in the Civil Procedure Code of 1908. It is guided by the principles of natural justice. These principles keep the CAT flexible in approach. Source: Indian Polity by Laxmikant- Chapter 60: Tribunals; Page no 60.1 and 60.2 46. Right to recall is a system of direct democracy. Which of the following is/are

demerits of the ‘system of recall’? (1) System of recall lessens the independence of elected representatives. (2) It makes the Council of Ministers more unstable. (3) It leads to policy paralysis of the government. Select the correct answer using the code given below: (a) 2 only (b) 1 and 2 only (c) 2 and 3 only (d) 1, 2 and 3

Answer: (d) Explanation: In simple terms, recall is the retiring of an elected officer by a vote of the electorate. This feature finds roots in the Athenian democracy and is also a prominent feature of the Constitution of Switzerland, Philippines and so on. In India, the bold initiative to demand the right to recall was taken by Jaiprakash Narayan in the 1974 during the Total Revolution movement Statement 1 is correct. Primarily, it leads to ‘excess’ of democracy and this will hamper the independence of the legislators. Statement 2 is correct. If right to recall becomes commonplace, it would only add to the instability of governments, by empowering not those who win elections, but those who lose. Statement 3 is correct. The legislators are expected to plan policies and schemes keeping in mind the greater good. With the recall in place, the legislators will tend to take fewer risks, not try out innovative policies fearing possible public outrage and eventual recall from the post. The legislators may tend to be afraid to take decisions, become lethargic and procrastinating in nature. It will eventually discourage them to take crucial decisions. With recall in place; it is hard to even think of a person who will take a decision in times of an imminent crisis Sources:

Vajiram & Ravi Institute yellow book, Part 2; Ch. 10: Election; Page No.132

https://www.thehindubusinessline.com/opinion/right-to-recall-a-dangerous-idea/Article20351844.ece1

47. Which of the following are the members of the Cabinet Committee on Economic

Affairs in India? (1) Minister of Home Affairs (2) Minister of External Affairs (3) Minister of Corporate Affairs

Page 33: Answer Key & Exp for GS Prelim Test-8 held on 20 …...1 Polity 2 Prelim IAS Test Series (2019) –GS Test 8 (20.01.2019) and Current Affairs (November 2018) Answer Key & Exp for GS

33

Prelim IAS Test Series (2019) – GS Test 8 (20.01.2019) Polity 2 and Current Affairs (November 2018)

(4) Minister of Defence Select the correct answer using the code given below: (a) 1 only (b) 1 and 3 only (c) 1, 2 and 3 only (d) 1, 2, 3 and 4

Answer: (d) Explanation: Composition of Cabinet Committee on Economic Affairs in India:

Prime Minister

Minister of Home Affairs

Minister of External Affairs

Minister of Finance

Minister of Corporate Affairs

Minister of Urban Development

Minister of Housing and urban poverty Alleviation

Minister of Information and Broadcasting

Minister of Defence

Minister of Railways

Minister of Chemical and Fertilizers

Minister of Parliamentary Affairs

Minister of Civil Aviation

Minister of Food Processing Industries Hence, option (d) is the correct answer. Source: http://cabsec.nic.in/cabinetcommittees.php 48. With respect to the provision of ‘Abolition of Untouchability’ under Article 17 of the

Indian Constitution, consider the following statements: (1) The term ‘Untouchability’ has been defined comprehensively in the Article 17

itself. (2) The Indian Parliament has enacted the Protection of Civil Rights Act to give

effect to the Article 17. Which of the statements given above is/are correct? (a) 1 only (b) 2 only (c) Both 1 and 2 (d) Neither 1 nor 2

Answer: (b) Explanation: Article 17 of the Constitution of India States that “Untouchability” is abolished and its practice in any form is forbidden. The enforcement of any disability arising out of Untouchability shall be an offence punishable in accordance with the law.

Page 34: Answer Key & Exp for GS Prelim Test-8 held on 20 …...1 Polity 2 Prelim IAS Test Series (2019) –GS Test 8 (20.01.2019) and Current Affairs (November 2018) Answer Key & Exp for GS

34

Prelim IAS Test Series (2019) – GS Test 8 (20.01.2019) Polity 2 and Current Affairs (November 2018)

Statement 1 is not correct: The term ‘Untouchability’ has not been defined either in the Indian Constitution or in the Protection of Civil Rights Act. However, the Mysore High Court held that the subject matter of Article 17 is not Untouchability in its literal or grammatical sense but the practice as it had developed historically in the country. It refers to the social disabilities imposed on certain classes of persons by reason of their birth in certain castes. Statement 2 is correct: In pursuance of the above Constitutional provision, the Untouchability (Offences) Act, 1955, was enacted. Subsequently, it was amended and renamed in the year 1976 as the "Protection of Civil Rights Act, 1955". Rules under this Act, viz, “The Protection of Civil Rights Rules, 1977” were notified in 1977. The Act extends to the whole of India and provides punishment for the practice of Untouchability. It is implemented by the respective State Governments and Union Territory Administrations. (Source: Indian Polity by M Laxmikanth- Chapter 7, page no. 7.8) 49. Consider the following statements about the Collective Privileges of the Indian

Parliament: (1) It can exclude strangers from its proceedings and hold secret sittings to

discuss some important matters. (2) The courts can’t inquire into the proceedings of both the Houses of the Indian

Parliament or their Committees. Which of the statements given above are correct? (a) 1 only (b) 2 only (c) Both 1 and 2 (d) Neither 1 nor 2

Answer: (c) Explanation: Statement 1 is correct: The 44th Amendment Act of 1978 restored the freedom of the press to publish true reports of Parliamentary proceedings without prior permission of the House. But this is not applicable in the case of a secret sitting of the House. It can exclude strangers from its proceedings and hold secret sittings to discuss some important matters. Statement 2 is correct: Under the collective privileges of the Indian Parliament, the courts (the Supreme Court and the High Courts) are prohibited to inquire into the proceedings of any House or its Committees. Source: Indian Polity by M. Laxmikanth, 5th edition- ch-22, Parliament, pg-22.31 50. With reference to ‘the State Information Commission’ in India, which of the following

statements is/are not correct? (1) The State Information Commissioners are appointed by the Governor of the

State concerned and can be removed by the President only. (2) The State Information Commissioners are eligible for reappointment. (3) The State Information Commission submits annual report to the respective

State Legislature on the implementation of the Right to Information (RTI) Act. Select the correct answer using the code given below:

Page 35: Answer Key & Exp for GS Prelim Test-8 held on 20 …...1 Polity 2 Prelim IAS Test Series (2019) –GS Test 8 (20.01.2019) and Current Affairs (November 2018) Answer Key & Exp for GS

35

Prelim IAS Test Series (2019) – GS Test 8 (20.01.2019) Polity 2 and Current Affairs (November 2018)

(a) 1 and 2 only (b) 2 and 3 only (c) 1 and 3 only (d) 1, 2 and 3

Answer: (b) Explanation: Statement 1 is correct: The State Information Commissioners are appointed by the Governor on the recommendation of a committee consisting of the Chief Minister as Chairperson, the Leader of Opposition in the Legislative Assembly and a State Cabinet Minister nominated by the Chief Minister. The Governor of the State concerned and not the President of India can remove the State Information Commissioners and the State Chief Information Commissioner from the office. He is removed in the manner similar to a Judge of the High Court on the ground of proved misbehaviour or incapacity. Statement 2 is not correct: The State Chief Information Commissioner and a State Information Commissioner hold the office for a term of 5 years or until they attain the age of 65 years, whichever is earlier. They are not eligible for reappointment. Statement 3 is not correct: The State Information Commission submits an annual report to the State Government on the implementation of the provisions of the Right to Information (RTI) Act. The State Government places this report before the State Legislature. Source: General Studies Indian Polity part-2; Vajiram & Ravi yellow book; Chapter-9 (Miscellaneous Topics); pg-114,115. 51. In the context of Bills in the Indian Parliament, consider the following statements:

(1) A Private Member Bill can be introduced by any Member of the Parliament after giving one month’s notice.

(2) The rejection of a Money Bill by the Lok Sabha may lead to the resignation of the Union Government.

Which of the statements given above is/are correct? (a) 1 only (b) 2 only (c) Both 1 and 2 (d) Neither 1 nor 2

Answer: (b) Explanation: Statement 1 is not correct: A Private Member Bill can be introduced by any Member of the Parliament other than a Minister. If a Minister introduces a Bill in the Indian Parliament then it is considered as a Public Bill. The introduction of a private Bill needs a notice of one month, while a Public Bill can be introduced in the Parliament on a notice of Seven days.

Page 36: Answer Key & Exp for GS Prelim Test-8 held on 20 …...1 Polity 2 Prelim IAS Test Series (2019) –GS Test 8 (20.01.2019) and Current Affairs (November 2018) Answer Key & Exp for GS

36

Prelim IAS Test Series (2019) – GS Test 8 (20.01.2019) Polity 2 and Current Affairs (November 2018)

Statement 2 is correct: Since a Money Bill is introduced in the House of the Parliament by a Minister, its rejection in the concerned House amounts to the expression of want of Parliamentary confidence in the Union Government and may lead to its resignation. Source: Indian Polity by M. Laxmikanth, 5th edition- ch-22, Parliament, pg-22.18 52. Which of the following is the objective of the application cVIGIL?

(a) To ensure timely implementation of projects and review by the Union Cabinet. (b) To enable a citizen to report bureaucratic corruption. (c) To allow citizens to report any violation of the Model Code of Conduct. (d) To provide vigilance in case of corruption in the Election Commission.

Answer: (c) Explanation: The Election Commission of India in July, 2018 launched a mobile app- cVIGIL - which citizens can use to confidentially report violation of election Code of Conduct. “cVIGIL” will allow anyone in the election-bound State to report violations of Model Code of Conduct (MCC) that comes into effect from the date of announcement of elections and goes on till a day after the polls. By using this app, citizens can immediately report on incidents of misconduct within minutes of having witnessed them and without having to rush to the office of the returning officer to lodge a complaint. The vigilant citizen has to click a picture or record a video of up to two minutes’ duration of the scene of violations of the Model Code. The photo or video is to be uploaded on the app. The automated location mapping will be done by the app using the Geographic Information System. After its successful submission through the app, the vigilant citizen gets a Unique ID to track and receive the follow up updates on her or his mobile. A citizen can report many incidents in this manner and will get a unique id for each report for follow up updates. The identity of the complainant will be kept confidential. The app has inbuilt features to prevent its misuse. It will receive complaints only about Model Code of Conduct violations. Sources:

http://pib.nic.in/newsite/PrintRelease.aspx?relid=180358

https://www.thehindu.com/news/cities/Hyderabad/with-cvigil-citizens-to-share-onus-of-enforcing-mcc/Article25290432.ece

53. With reference to ‘the Motions in the Indian Parliament’, consider the following

statements: (1) A Censure motion or No-Confidence Motion need not state the reasons for its

adoption in the Lok Sabha. (2) A No-Confidence Motion can be moved only against the entire Union Council

of Ministers unlike a Censure Motion, which can be moved against a Minister also.

(3) Only the Lok Sabha can remove the entire Union Council of Ministers by passing a No-Confidence Motion.

Which of the statements given above are correct? (a) 1 and 2 only

Page 37: Answer Key & Exp for GS Prelim Test-8 held on 20 …...1 Polity 2 Prelim IAS Test Series (2019) –GS Test 8 (20.01.2019) and Current Affairs (November 2018) Answer Key & Exp for GS

37

Prelim IAS Test Series (2019) – GS Test 8 (20.01.2019) Polity 2 and Current Affairs (November 2018)

(b) 1 and 3 only (c) 2 and 3 only (d) 1, 2 and 3

Answer: (c) Explanation Statement 1 is not correct: A No-Confidence motion need not state the reason for its adoption in the Lok Sabha whereas a Censure Motion should state the reasons for its adoption in the Lok Sabha. Statement 2 is correct: A No-Confidence Motion in the Lok Sabha can be moved against the entire Union Council of Ministers only and not against an individual Minister of Parliament. It is moved for ascertaining the confidence of Lok Sabha in the Union Council of Ministers. Whereas a Censure motion can be moved against an individual Minister or a group of Ministers or the entire Union Council of Ministers. It is moved for censuring the Council of Ministers for specific policies and actions. Statement 3 is correct: If a censure motion is passed in the Lok Sabha, the council of Minister need not resign from their offices, but if a No-Confidence Motion is passed in the Lok Sabha, the Union Council of Ministers must resign from office( as it is collectively responsible to the Lok Sabha under Article 75 of the Constitution). A no confidence motion needs the support of 50 members of the Lok Sabha to be admitted. Source: M. Indian Polity by Laxmikanth, 5th edition- ch-22, Parliament, pg-22.16 54. With reference to the National Development Council (NDC) in India, consider the

following statements: (1) NDC is responsible for policy matters with regard to planning for social and

economic development. (2) The recommendations of NDC are binding on the Government of India. Which of the statements given above is/are not correct? (a) 1 only (b) 2 only (c) Both 1 and 2 (d) Neither 1 nor 2

Answer: (b) Explanation: Statement 1 is correct: The NDC is a forum of the discussion not only for plans and programmes but also for the social and economic matters of national importance. Statement 2 is not correct: The prime function of the National Development Council is to act as a bridge between the Union Government, and the State Governments. However, it is listed as an advisory body to the NITI Aayog and it’s recommendations are not binding. Source: Indian Polity by M. Laxmikant, Ch. 52, NITI Aayog, Page 52.11 55. Regarding the ‘veto power of the President of India’, which of the following features

are correct? (1) It can be exercised over a Constitutional Amendment Bill.

Page 38: Answer Key & Exp for GS Prelim Test-8 held on 20 …...1 Polity 2 Prelim IAS Test Series (2019) –GS Test 8 (20.01.2019) and Current Affairs (November 2018) Answer Key & Exp for GS

38

Prelim IAS Test Series (2019) – GS Test 8 (20.01.2019) Polity 2 and Current Affairs (November 2018)

(2) The President can withhold his assent to a Money Bill. (3) The State legislatures cannot override the veto power of the President. Select the correct answer using the code given below: (a) 1 and 2 only (b) 1 and 3 only (c) 2 and 3 only (d) 1, 2 and 3 only

Answer: (c) Explanation: The President has the veto power over the Bills passed by the Parliament and State Legislatures. Statement 1 is not correct. It cannot be exercised over a Constitutional Amendment Bill. Statement 2 is correct. The President can withhold his assent to a Money Bill but cannot return it for reconsideration of the Houses of the Parliament. Statement 3 is correct. The State legislatures cannot override the veto power of the President. If the Bill is passed again by the State legislature with or without amendments and presented again to the President for his assent, the President is not bound to give his assent to the Bill. Hence, the correct answer is option(c). Source: Indian Polity by M. Laxmikanth; 5th edition, Chapter 17 Page 358 56. Paid news vitiates the concept of free and fair election in a democratic polity of

India. In this context, which of the following statements is/are correct? (1) Paid news is an electoral offence under the Representation of People Act,

1951. (2) The Election Commission of India has constitutionally mandated powers to

regulate paid news. Select the correct answer using the code given below: (a) 1 only (b) 2 only (c) Both 1 and 2 (d) Neither 1 nor 2

Answer: (d) Explanation: Paid news or paid content are those Articles in newspapers, magazines and the electronic media, which indicate favourable conditions for the Institution which has paid for the content. The news is much like an advertisement but without the ad tag. This kind of news has been considered a serious malpractice since it deceives the citizens, not letting them know that the news is, in fact, an advertisement. Statement 1 is not correct. Paid news in itself is not an electoral offence in realm of Representation of People Act, 1951. However, if expenditure by candidates on paid news gets established and candidates has not reported or underreported any such expenditure, the act in such cases becomes electoral offence. However, Law commission in its

Page 39: Answer Key & Exp for GS Prelim Test-8 held on 20 …...1 Polity 2 Prelim IAS Test Series (2019) –GS Test 8 (20.01.2019) and Current Affairs (November 2018) Answer Key & Exp for GS

39

Prelim IAS Test Series (2019) – GS Test 8 (20.01.2019) Polity 2 and Current Affairs (November 2018)

report in 2015 has recommended making Paid News an electoral offence under the Representation of People Act, 1951. Statement 2 is not correct. Election Commission of India doesn’t have constitutionally mandated powers to regulate paid news. Additional Information: (1) In 2017, the Election Commission found a way around the lack of a specific law on

paid news and disqualified Narottam Mishra and barred him from contesting elections for 3 years by using Section 10A of the Representation of the People Act, 1951 that deals with misreporting of funds.

(2) The disqualification order, however was set aside by the Delhi High Court which argued that news stories could not be regulated by the Election Commission since they fall under the domain of free speech.

(3) Later on the Supreme Court stayed the Delhi High Court verdict, which at present leaves the Election Commission free to take up such cases of paid news in both the forthcoming elections as well as general elections in May 2019.

Sources:

Vajiram & Ravi Institute yellow book, Part 2; Ch. 10: Election; Page No.132

https://economictimes.indiatimes.com/news/politics-and-nation/paid-news-ec-may-challenge-delhi-hc-order-on-disqualification-of-bjps-narottam mishra/Articleshow/65191575.cms

57. With reference to ‘the Ordinance making power of the President under Article 123 of

the Indian Constitution’, which of the following features is/are correct? (1) It can be issued to amend the Constitution. (2) It can be given effect retrospectively. (3) It has been borrowed from the US Constitution. (4) Maximum life of an Ordinance is six months. Select the correct answer using the code given below: (a) 2 only (b) 1 and 2 only (c) 2 and 3 only (d) 1, 3 and 4 only

Answer: (a) Explanation: Article 123 of the Indian Constitution provides power to the President to promulgate Ordinances when either of the Houses of Parliament is not in session. Feature 1 is not correct. It may modify or repeal any act of Parliament or another Ordinance. It can alter or amend a tax law also. However, it cannot be issued to amend the Constitution. Feature 2 is correct. An Ordinance like any other legislation, can be retrospective, that is, it may come into force from a back date. Feature 3 is not correct. The Ordinance-making power of the President in India is rather unusual and not found in most of the democratic Constitutions of the world including that of USA and UK.

Page 40: Answer Key & Exp for GS Prelim Test-8 held on 20 …...1 Polity 2 Prelim IAS Test Series (2019) –GS Test 8 (20.01.2019) and Current Affairs (November 2018) Answer Key & Exp for GS

40

Prelim IAS Test Series (2019) – GS Test 8 (20.01.2019) Polity 2 and Current Affairs (November 2018)

Feature 4 is not correct. The maximum life of an Ordinance can be six months and six weeks, in case of non-approval by the Parliament (six months being the maximum gap between the two sessions of Parliament). Source: Indian Polity by M. Laxmikanth; 5th edition, Chapter 17 Page 17.11 58. With reference to the special provisions for Sikkim in the Indian Constitution,

consider the following: (1) The minimum strength of Sikkim Legislative Assembly should be thirty. (2) The President of India can extend any law to Sikkim which is in force in a

State of the Indian Union. Which of the statements given above is/are correct? (a) 1 only (b) 2 only (c) Both 1 and 2 (d) Neither 1 nor 2

Answer: (c) Explanation: Sikkim joined the mainstream of national life with the merger of the State with India in the year 1975. By virtue of Parliament’s 36th Constitutional Amendment, the State became the 22nd State of India. Article 371 F was inserted into the Constitution of India vide the 36th Constitutional Amendment Act on the 16th of May 1975. It provides special provision for the State of Sikkim. Statement 1 is correct. As per Article 371 F(a), the Legislative Assembly of the State of Sikkim shall consist of not less than thirty members. Statement 2 is correct. As per Article 371 F(n), the President can extend to Sikkim any law which is in force in a State of the Indian Union. Source: Vajiram & Ravi Institute Yellow book, Part 2; Ch. 4: Union Territories; Page No. 65 59. With reference to the Speaker of the Legislative Assembly in India, consider the

following statements: (1) He has only the casting vote in case of a tie of votes on any matter in the

State Legislative Assembly. (2) He decides on disqualification of members of the State Legislative Assembly,

arising on grounds of defection. (3) He is the final interpreter of the Constitution of India within the State

Legislative Assembly. Which of the statements given above is/are correct? (a) 1 and 3 only (b) 3 only (c) 2 and 3 only (d) 1, 2 and 3

Answer: (d)

Page 41: Answer Key & Exp for GS Prelim Test-8 held on 20 …...1 Polity 2 Prelim IAS Test Series (2019) –GS Test 8 (20.01.2019) and Current Affairs (November 2018) Answer Key & Exp for GS

41

Prelim IAS Test Series (2019) – GS Test 8 (20.01.2019) Polity 2 and Current Affairs (November 2018)

Explanation: Statement 1 is correct. A casting vote is a vote that someone may exercise to resolve a deadlock. A casting vote is typically exercised by the presiding officer of a Legislative Council, Legislative Assembly, Committee of the State Legislature etc., and may only be exercised to break a deadlock. Under the Indian Constitution, the Speaker exercises only a casting vote in the case of equality of votes in any matter of the State Legislature. Statement 2 is correct. Anti-defection legislation was made way back in 1985 through the 52nd amendment to the Constitution and incorporating it in the 10th schedule with a genuine view that graft and money power will diminish in politics. The Tenth Schedule of the Indian Constitution says the Speaker's/Chairperson's decision on questions of disqualification on ground of defection shall be final and can't be questioned in courts. In Kihoto Hollohan vs Zachillhu and others (1991) case the Supreme Court Constitution Bench declared that the Speaker's decision was subject to judicial review. Statement 3 is correct. Speaker is the final interpreter of the Constitution of India, rules of procedure and conduct of business of assembly, the legislative precedence within the assembly. Sources: Indian Polity by M. Laxmikanth 4th Edition, Ch 29, State Legislature; Page no. 29.5 60. Which among the following States in India never had a State Legislative Council?

(a) West Bengal (b) Odisha (c) Tamil Nadu (d) Maharashtra

Answer: (b) Explanation: West Bengal had Legislative Council. The Council was abolished on 21 March 1969. Subsequently; the Parliament passed the West Bengal Legislative Council (Abolition) Act, 1969 with effect from August 1, 1969. Odisha never had a State Legislative Council. The Odisha Assembly passed a resolution for establishing Legislative Council in August, 2018. Hence, option (a) is the correct answer. Tamil Nadu had Legislative Council. The Council was abolished in 1986. The State of Maharashtra has upper House of the State legislature i.e. Legislative Council. Sources:

https://www.thehindu.com/news/national/other-States/trinamool-to-revive-legislative-council-in-wb/Article2112981.ece

https://www.thehindu.com/news/national/other-States/odisha-House-nod-for-legislative-council/Article24886709.ece

https://www.thehindu.com/news/national/tamil-nadu/Legislative-Council-had-chequered-history/Article16364791.ece

Page 42: Answer Key & Exp for GS Prelim Test-8 held on 20 …...1 Polity 2 Prelim IAS Test Series (2019) –GS Test 8 (20.01.2019) and Current Affairs (November 2018) Answer Key & Exp for GS

42

Prelim IAS Test Series (2019) – GS Test 8 (20.01.2019) Polity 2 and Current Affairs (November 2018)

61. Consider the following statements with respect to the establishment of Tribunals in India: (1) Under Article 323A, tribunals can only be established by the Parliament of

India for the administrative purposes. (2) Under Article 323B, the Parliament and the State Legislatures are authorized

to provide for the establishment of tribunals for the adjudication of disputes related to taxation and land reforms.

Which of the statements given above is/are correct? (a) 1 only (b) 2 only (c) Both 1 and 2 (d) Neither 1 nor 2

Answer: (c) Explanation: Statement (1) is correct: While tribunals under Article 323A can be established only by Parliament, tribunals under Article 323B can be established both by Parliament and State Legislatures with respect to matters falling within their legislative competence. Article 323A contemplates establishment of tribunals for public service matters only. Statement (2) is correct: Under Article 323B, the Parliament and the State Legislature are authorized to provide for the establishment of tribunals for the adjudication of disputes relating to the following matter:

Taxation

Foreign exchange

Industrial and labour

Land reforms

Ceiling on urban property

Elections to Parliament and State legislatures

Food stuffs

Rent and tenancy rights Source: Polity by Laxmikant- Chapter 60: Tribunals; Page no 60.2 62. With reference to the provisions regarding powers of the President of India with

respect to Jammu and Kashmir, which of the following statements is not correct? (a) The President has no power to declare a Financial Emergency in relation to

the State. (b) A National Emergency declared on the ground of internal disturbance will not

have effect in the State except with the concurrence of the State government. (c) The President has no power to suspend the Constitution of the State on the

ground of failure to comply with the directions given by him. (d) President’s Rule on the ground of failure of the constitutional machinery under

the Indian Constitution is not applicable to the State. Answer: (d) Explanation:

Page 43: Answer Key & Exp for GS Prelim Test-8 held on 20 …...1 Polity 2 Prelim IAS Test Series (2019) –GS Test 8 (20.01.2019) and Current Affairs (November 2018) Answer Key & Exp for GS

43

Prelim IAS Test Series (2019) – GS Test 8 (20.01.2019) Polity 2 and Current Affairs (November 2018)

Following are the special provisions of the Indian Constitution with respect to Jammu and Kashmir:

The President has no power to declare a financial emergency in relation to the State.

A National Emergency declared on the ground of internal disturbance will not have effect in the State except with the concurrence of the State Government.

The President has no power to suspend the Constitution of the State on the ground of failure to comply with the directions given by him.

President Rule on the ground of failure of Constitutional machinery under the provisions of Constitution of Jammu and Kashmir is applicable to the State. However notably, in 1964, Article 356 of the Indian Constitution (dealing with the imposition of President’s Rule in a State) was extended to the State of J&K.

Hence, option(d) is the correct answer. Source: Indian Polity by M. Laxmikanth; 5th edition, Chapter 36, Pg. 36.3-36.5 63. Which of the following Parliamentary committees are chaired by the Speaker of the

Lok Sabha? (1) Public Accounts Committee (2) Business Advisory Committee (3) Ethics Committee (4) Rules Committee (5) General Purpose Committee Select the correct answer using the codes given below: (a) 1 and 3 only (b) 2 and 5 only (c) 1, 3 and 4 only (d) 2, 4 and 5 only

Answer: (d) Explanation:

PUBLIC ACCOUNTS COMMITTEE: The function of the committee is to examine the annual audit reports of the Comptroller and Auditor General of India which are laid before the Parliament by the President. At present it consists of 22 members; 15 from Lok Sabha and 7 from Rajya Sabha. The members are elected by the Parliament every year from amongst its members accordingly to the principle of proportional representation by means of the single transferable vote. Chairman of the committee is selected from the Opposition and appointed by the Speaker.

BUSINESS ADVISORY COMMITTEE: It regulates the programme and time table of the House. The Lok Sabha Committee consists of 15 members including the Speaker as its Chairman. In Rajya Sabha, it has 11 members including the Chairman as its ex-officio Chairman.

ETHICS COMMITTEE: It enforces the code of conduct of members of Parliament. It examines the cases of misconduct and recommends appropriate action. Thus, it is engaged in maintaining discipline and decorum in Parliament. The Speaker is not a member of this Committee.

Page 44: Answer Key & Exp for GS Prelim Test-8 held on 20 …...1 Polity 2 Prelim IAS Test Series (2019) –GS Test 8 (20.01.2019) and Current Affairs (November 2018) Answer Key & Exp for GS

44

Prelim IAS Test Series (2019) – GS Test 8 (20.01.2019) Polity 2 and Current Affairs (November 2018)

RULES COMMITTEE: It considers the matters of procedure and conduct of business in the House and recommends necessary amendments or additions to the Rules of the House. The Lok Sabha committee consists of 15 members including the Speaker as its ex-officio Chairman. In Rajya Sabha, it consists of 16 members including the Chairman as its ex-officio Chairman.

GENERAL PURPOSES COMMITTEE: It considers and advises on matters concerning affairs of the House, which do not fall within the jurisdiction of any other Parliamentary committee. In each House, the committee consists of the presiding officer (Speaker/Chairman) as its ex-officio Chairman, Deputy Speaker (Deputy Chairman of the Rajya Sabha), Chairpersons of all the departmental standing committees of the House, leaders of recognized parties and groups in the House and such other members as nominated by the presiding officer.

Hence, option (d) is the correct answer. Source: Indian Polity by Laxmikanth- Chapter 23: Parliamentary Committee; Page no 23.8 and 23.9 64. With reference to the Calling Attention Motion in the Indian Parliament, consider the

following statements: (1) It is necessary for the Chairman of the Rajya Sabha to admit Calling Attention

Motion for every sitting of the House. (2) Calling Attention Motion has to be taken up immediately after the Question

Hour and laying of the papers. (3) More than one matter shall be raised at the same meeting of the House, when

Calling Attention Motion is taken up. Which of the statements given above is/are not correct? (a) 1 and 2 only (b) 1 and 3 only (c) 2 and 3 only (d) 1, 2 and 3

Answer: (d) Explanation: Statement (1) is not correct: It is not necessary for the Chairman of the Rajya Sabha to admit the Calling Attention notice for every sitting of the House. Admission of notice is subject to the rules and ultimately to the decision of the Chairman that the matter sought to be raised does call for an early Statement from the Minister. Statement (2) is not correct: Generally, the ‘Calling Attention’ is taken up immediately after the Question Hour and laying of papers, if any, but sometimes the Calling Attention may be taken up later in the day, subject to the direction of the Chair and the consensus arrived at in the House. Statement (3) is not correct: Not more than one matter shall be raised at the same sitting of admission of Calling Attention Motion. In the event of more than one matter being presented for the same day, priority shall be given to the matter which is, in the opinion of the Chairman of Rajya Sabha, more urgent and important. Source: https://rajyasabha.nic.in/rsnew/practice_procedure/book8.asp

Page 45: Answer Key & Exp for GS Prelim Test-8 held on 20 …...1 Polity 2 Prelim IAS Test Series (2019) –GS Test 8 (20.01.2019) and Current Affairs (November 2018) Answer Key & Exp for GS

45

Prelim IAS Test Series (2019) – GS Test 8 (20.01.2019) Polity 2 and Current Affairs (November 2018)

65. Regarding the famous “Objectives Resolution” proposed by Jawahar Lal Nehru,

consider the following statements: (1) It was proposed to the Constituent Assembly of India before the enactment of

the Constitution. (2) It contained clear provisions to abolish Privy Purse from India. Select the correct answer using the code given below: (a) 1 only (b) 2 only (c) Both 1 and 2 (d) Neither 1 nor 2

Answer: (a) Explanation: Statement (1) is correct: It was proposed in 1946, and later passed by the Constituent Assembly shaping the Preamble of the Indian Constitution. Statement (2) is not correct: There were no such provisions to abolish Privy Purse from India. Source: Indian Polity by Laxmikant- Chapter 2: Making of the Constitution; Page no 2.2 and Chapter 4: Preamble of the Constitution; Page no 4.2 66. Which of the following statements is/are correct about ‘mutual delegation of

administrative functions between the Union and the State Governments in India? (1) The mutual delegation of administrative function by the President of India and

the Governor of the State concerned is unconditional. (2) The President can entrust executive functions of the Union/Centre to the State

concerned without the consent of the State. Select the correct answer using the code given below: (a) 1 only (b) 2 only (c) Both 1 and 2 (d) Neither 1 nor 2

Answer: (d) Explanation: Statement (1) is not correct: The Mutual delegation of Administrative functions by the President of India and Governor of the concerned State may be conditional or unconditional. Statement (2) is not correct: The Constitution of India makes provision for the entrustment of the executive functions of the Centre to a State without the consent of that State. In this case, the delegation is done by the Parliament and not by the President. Source: Indian Polity by Laxmikanth- Chapter 14: Centre-State Relations; Page no 14.5

Page 46: Answer Key & Exp for GS Prelim Test-8 held on 20 …...1 Polity 2 Prelim IAS Test Series (2019) –GS Test 8 (20.01.2019) and Current Affairs (November 2018) Answer Key & Exp for GS

46

Prelim IAS Test Series (2019) – GS Test 8 (20.01.2019) Polity 2 and Current Affairs (November 2018)

67. Regarding the powers and functions of the Vice President of India, which of the following statements is/are correct? (1) Like in the United States of America, the Indian Vice-President succeeds to

the Presidency when it falls vacant. (2) In case of any delay in conducting the election of new President of India by

any reason, the Vice-President does not get the opportunity to act as the President.

Select the correct answer using the code below: (a) 1 only (b) 2 only (c) Both 1 and 2 (d) Neither 1 nor 2

Answer: (b) Explanation: The election and impeachment of the President are performed by two slightly different set of people. Statement 1 is not correct: The American Vice-President succeeds to the presidency when it falls vacant, and remains President for the unexpired term of his predecessor. The Indian Vice-President, on the other hand, does not assume the office of the President when it falls vacant for the unexpired term. He merely serves as an acting President until the new President assumes charge. Statement 2 is correct: When the vacancy is going to be caused by the expiration of the term of the sitting President, an election to fill the vacancy must be held before the expiration of the term. In case of any delay in conducting the election of new President by any reason, the outgoing President continues to hold office (beyond his term of five years) until his successor assumes charge. This is provided by the Constitution in order to prevent an ‘interregnum’. In this situation, the Vice-President does not get the opportunity to act as the President or to discharge the functions of the President. Source: Indian Polity by M. Laxmikanth; 5th edition; Chapter 18 Page 18.3 68. Which of the following rights/freedom are protected under Article 19 of the

Constitution of India? (1) The freedom of silence (2) The right to move out of the country (3) The freedom of commercial advertisement (4) The right to strike Select the correct answer using the code given below: (a) 1 only (b) 1 and 3 only (c) 1, 2 and 3 only (d) 1, 2, 3 and 4

Answer: (b) Explanation:

Page 47: Answer Key & Exp for GS Prelim Test-8 held on 20 …...1 Polity 2 Prelim IAS Test Series (2019) –GS Test 8 (20.01.2019) and Current Affairs (November 2018) Answer Key & Exp for GS

47

Prelim IAS Test Series (2019) – GS Test 8 (20.01.2019) Polity 2 and Current Affairs (November 2018)

The Supreme Court held that the freedom of speech and expression under Article 19 of the Constitution includes the following: Right to propagate one’s views as well as views of others. Freedom of the press. Freedom of commercial advertisements. Freedom of silence. Right against imposition of pre-censorship on a newspaper. Right to demonstration or picketing but not right to strike.

The freedom of movement has two dimensions, viz, internal (right to move inside the country) and external (right to move out of the country and right to come back to the country). Article 19 protects only the first dimension. The second dimension is dealt by Article 21(right to life and personal liberty).

Freedom of Assembly: Every citizen has the right to assemble peacefully and without arms. It includes the right to hold public meetings, demonstrations and take out processions. This freedom can be exercised only on public land and the assembly must be peaceful and unarmed. This provision does not protect violent, disorderly, riotous assemblies, or one that causes breach of public peace or one that involves arms. This right does not include the right to strike.

Hence, option (b) is the correct answer. (Source: Indian Polity by M Laxmikanth- Ch. 7: Fundamental Rights, page no. 7.9-7.10) 69. The President shall cause to be published under his authority the translation of

Indian Constitution in Hindi language. Consider the following statements in this context: (1) This provision was brought by the 58th Constitutional Amendment Act of 1987. (2) If any discrepancy arises in this matter, the Parliament shall cause the Hindi

text to be revised suitably. Which of the statements given above is/are correct? (a) 1 only (b) 2 only (c) Both 1 and 2 (d) Neither 1 nor 2

Answer: (a) Explanation: The Constitution of India did not make any provision with respect to an authoritative text of the Constitution in the Hindi language. Later, a provision in this regard was made by the 58th Constitutional Amendment Act of 1987. This amendment inserted a new Article 394A in Part XXII. Hence, Statement (1) is correct. These Articles contain the following provisions: 1. The translation of the Constitution and its every amendment published shall be

deemed to be, for all purposes, its authoritative text in Hindi. 2. The President shall cause to be published under his authority:

Page 48: Answer Key & Exp for GS Prelim Test-8 held on 20 …...1 Polity 2 Prelim IAS Test Series (2019) –GS Test 8 (20.01.2019) and Current Affairs (November 2018) Answer Key & Exp for GS

48

Prelim IAS Test Series (2019) – GS Test 8 (20.01.2019) Polity 2 and Current Affairs (November 2018)

(a) The translation of the Constitution in Hindi language. The modifications which are necessary to bring it in conformity with the language, style and terminology adopted in the authoritative texts of the Central Acts in Hindi can be made in it.

(b) The translation in Hindi of every amendment of the Constitution made in English. 3. The translation of the Constitution and its every amendment published shall be

construed to have the same meaning as the original text in English. If any difficulty arises in this matter, the President shall cause the Hindi text to be revised suitably. Hence, Statement (2) is not correct.

Source: Polity by Laxmikant - Chapter 62: Authoritative Text of the Constitution in Hindi Language, Page no 62.1 70. Consider the following pairs:

Zonal Council State

1. Northern Zonal Council : Rajasthan

2. Central Zonal Council : Uttarakhand

3. Eastern Zonal Council : Sikkim

4. Western Zonal Council : Madhya Pradesh

5. Southern Zonal Council : Andhra Pradesh

Which of the pair(s) given above is/are not correctly matched? (a) 2 only (b) 3 only (c) 3 and 4 only (d) 2, 3 and 4 only

Answer: (c) Explanation: The Zonal Councils are the statutory bodies. They are established by the States Reorganization Act of 1956. The act divided the country into five zones:

The Northern Zonal Council, comprising the States of Haryana, Himachal Pradesh, Jammu & Kashmir, Punjab, Rajasthan, National Capital Territory of Delhi and Union Territory of Chandigarh;

The Central Zonal Council, comprising the States of Chhattisgarh, Uttarakhand, Uttar Pradesh and Madhya Pradesh;

The Eastern Zonal Council, comprising the States of Bihar, Jharkhand, Orissa, Sikkim and West Bengal;

The Western Zonal Council, comprising the States of Goa, Gujarat, Maharashtra and the Union Territories of Daman & Diu and Dadra & Nagar Haveli;

The Southern Zonal Council, comprising the States of Andhra Pradesh, Karnataka, Kerala, Tamil Nadu and the Union Territory of Puducherry.

The North Eastern States i.e. (i) Assam (ii) Arunachal Pradesh (iii) Manipur (iv) Tripura (v) Mizoram (vi) Meghalaya and (vii) Nagaland are not included in the Zonal Councils and their special problems are looked after by the North Eastern Council, set up under the

Page 49: Answer Key & Exp for GS Prelim Test-8 held on 20 …...1 Polity 2 Prelim IAS Test Series (2019) –GS Test 8 (20.01.2019) and Current Affairs (November 2018) Answer Key & Exp for GS

49

Prelim IAS Test Series (2019) – GS Test 8 (20.01.2019) Polity 2 and Current Affairs (November 2018)

North Eastern Council Act, 1972. The State of Sikkim has also been included in the North Eastern Council vide North Eastern Council (Amendment) Act, 2002 notified on 23rd December 2002. Consequently, action for exclusion of Sikkim as member of Eastern Zonal Council has been initiated by Ministry of Home Affairs. Hence, option(c) is the correct answer. Source: https://mha.gov.in/zonal-council 71. Which of the following Committees recommended for the appointment of the

Finance Commission for the Panchayats? (a) Balwant Rai Mehta Committee (b) G.V.K Rao Committee (c) Ashok Mehta Committee (d) L. M. Singhvi Committee

Answer: (d) Explanation: A Committee led L. M. Singhvi was constituted in the 1980s to recommend ways to revitalize PRIs. The Gram Sabha was considered as the base of a decentralisation, and PRIs viewed as institutions of self-governance which would actually facilitate the participation of the people in the process of planning and development. The committee recommended that: 1. The Panchayati Raj Institutions should be constitutionally recognized and

protected. New chapter in the Constitution should be provided to define their powers and functions.

2. Free and fare elections to be conducted through the Election Commission. 3. The appointment of Finance Commission for them. 4. All the rural development programmes to be entrusted to the Panchayati Raj

Institutions by amending Schedule VII to the Constitution. Hence, option (d) is the correct answer. Source: Vajiram & Ravi Institute yellow book, Part 2; Ch. 3, page-45 72. With reference to the Lok Adalats in India, which of the following statements is

correct? (a) Lok Adalats have the jurisdiction to settle matters at pre-litigation stage only. (b) Lok adalats can deal with matters of both civil and criminal in nature. (c) Every Lok Adalat consists of either serving or retired judicial officers only. (d) Lok Adalats decide the matters before it strictly according to the procedure

established by law. Answer: (b) Explanation:

Option (a) is not correct: Cases that are pending in regular courts can also be transferred to a Lok adalat if both the parties agree. Hence, Lok Adalats have the jurisdiction to settle matters at both the pre-litigation and post-litigation stage.

Option (b) is correct: Lok Adalats can deal with any matter falling within the jurisdiction whether it is of civil or criminal in nature, such as cheque-bouncing cases.

Page 50: Answer Key & Exp for GS Prelim Test-8 held on 20 …...1 Polity 2 Prelim IAS Test Series (2019) –GS Test 8 (20.01.2019) and Current Affairs (November 2018) Answer Key & Exp for GS

50

Prelim IAS Test Series (2019) – GS Test 8 (20.01.2019) Polity 2 and Current Affairs (November 2018)

Option (c) is not correct: Lok Adalats are usually presided over by retired judges, social activists, or other members of the legal profession.

Option (d) is not correct: The procedural laws and the Evidence Act are not strictly followed while assessing the merits of the claim by the Lok Adalat.

Source: Indian Polity by M. Laxmikant, Ch. 29, Public Interest Litigation, Page 29.1 73. The Parliament of India can make laws on any subject given under the three lists of

the Seventh Schedule for the Union Territories. In this context, Legislative Assembly of Puducherry can make laws on: (1) State List (2) Concurrent List (3) State List except land, police and public order (4) Concurrent List except land, police and public order Select the correct answer using the code given below: (a) 1 and 2 only (b) 3 and 4 only (c) 1, 2 and 3 only (d) 1, 2, 3 and 4

Answer: (a) Explanation:

The Parliament can make laws on any subject of the three lists (including the State List) for the Union Territories. This power of the Parliament also extends to Union Territories of Puducherry and Delhi, which have their own local legislatures.

This means that, the legislative power of the Parliament for the Union Territories on subjects of the State List remain unaffected even after establishing a Legislative Assembly for them.

The Legislative Assembly of Puducherry can also make laws on any subject of the State List and the Concurrent List.

Similarly, the Legislative Assembly of Delhi can make laws on any subject of the State List (except public order, police and land) and the Concurrent List.

Hence, option(a) is the correct answer. Source: Vajiram & Ravi Yellow Book, Part-2; Chapter-3, page-58. 74. Regarding elections of the members of Panchayats in India, consider the following

statements: (1) All the Panchayat seats in village shall be filled by persons chosen by direct

election. (2) Minimum age to contest elections in Panchayat is 25 years. Which of the statements given above is/are correct? (a) 1 only (b) 2 only (c) Both1 and 2 (d) Neither 1 nor 2

Page 51: Answer Key & Exp for GS Prelim Test-8 held on 20 …...1 Polity 2 Prelim IAS Test Series (2019) –GS Test 8 (20.01.2019) and Current Affairs (November 2018) Answer Key & Exp for GS

51

Prelim IAS Test Series (2019) – GS Test 8 (20.01.2019) Polity 2 and Current Affairs (November 2018)

Answer: (a) Explanation:

Statement 1 is correct- All the seats in a Panchayat shall be filled by persons chosen by direct elections from territorial constituencies in the Panchayat area i.e. a village/villages.

Statement 2 is not correct- The minimum age for contesting elections to the Panchayats is 21 years and not 25 years.

Source: Vajiram & Ravi Institute yellow book, Part 2; Ch. 3, page-46 75. Which of the following provisions are amended by special majority in the Indian

Parliament? (1) Creation/Abolition of a State Legislative Council (2) Fundamental Rights (3) Election of the President (4) Directive Principles of State Policy (5) Formation of new States Select the correct answer using the code given below: (a) 1, 2 and 4 only (b) 2 and 4 only (c) 1, 2, 3 and 4 only (d) 1, 2, 3, 4 and 5

Answer: (b) Explanation: The Indian Constitution can be amended in three ways: A. Amendment by simple majority of the Parliament, B. Amendment by special majority of the Parliament, and C. Amendment by special majority of the Parliament and the ratification of half of the

State legislatures. Special Majority of Parliament: The majority of the provisions in the Constitution need to be amended by a special majority of the Parliament, i.e. 50% of the total membership of each House and a majority of two-thirds of the members of each House present and voting. Following provisions can be amended by this way:

Fundamental Rights;

Directive Principles of State Policy; and

All other provisions which are not covered by the simple majority and by special majority with the consent of States.

Hence, (b) is the correct answer. (Source: Indian Polity by M Laxmikanth- 4th edition Chapter 10, page no. 10.2-10.3) 76. Consider the following statements:

(1) India became a secular State in 1976 with the 42nd Constitutional Amendment Act.

Page 52: Answer Key & Exp for GS Prelim Test-8 held on 20 …...1 Polity 2 Prelim IAS Test Series (2019) –GS Test 8 (20.01.2019) and Current Affairs (November 2018) Answer Key & Exp for GS

52

Prelim IAS Test Series (2019) – GS Test 8 (20.01.2019) Polity 2 and Current Affairs (November 2018)

(2) Secularism, in the Indian context, provides for the irreligious nature of the State.

Which of the statements given above is/are correct? (a) 1 only (b) 2 only (c) Both 1 and 2 (d) Neither 1 nor 2

Answer: (d) Explanation: Statement 1 is not correct. The term ‘secularism’ was indeed added into Preamble of the Indian Constitution 1976 through 42nd Constitutional Amendment Act. However the Indian Constitution was secular in nature since its inception. In 1974 Supreme Court said that, although the phrase ‘secular State’ was not explicitly mentioned in the Constitution, there can be no doubt that the Constitution-makers wanted to establish such a State and accordingly Articles 25 to 28 (guaranteeing the fundamental right to freedom of religion) have been included in the Constitution. Also, the Indian Constitution never provided for any State religion. Statement 2 is not correct. The Indian Constitution ensures that State doesn’t have any religion of its own. It is neither religious nor irreligious nor anti-religious. The Indian Constitution embodies the positive concept of secularism i.e. all religions in our country have the same status and support from the State. Sources:

Vajiram & Ravi Yellow Book, Indian Polity, Part 1, Page 10

Indian Polity, M Laxmikanth, 5th Edition, Ch.4: Preamble of the Constitution Page 4.4

77. In Indian context, consider the following statements about ‘the Citizenship

(Amendment) Bill, 2016’: (1) The Bill amends the Citizenship Act, 1955 to make illegal migrants who are

Hindus, Sikhs, Buddhists, Jains, Parsis and Christians from Afghanistan, Bangladesh and Pakistan, eligible for citizenship.

(2) The Bill excludes illegal migrants for citizenship on the basis of religion. (3) The Bill allows cancellation of Overseas Citizens of India (OCI) registration for

violation of any law. Which of the statements given above is /are correct? (a) 1 only (b) 2 and 3 only (c) 1 and 3 only (d) 1, 2 and 3

Answer: (c) Explanation: Highlights of the Bill

Page 53: Answer Key & Exp for GS Prelim Test-8 held on 20 …...1 Polity 2 Prelim IAS Test Series (2019) –GS Test 8 (20.01.2019) and Current Affairs (November 2018) Answer Key & Exp for GS

53

Prelim IAS Test Series (2019) – GS Test 8 (20.01.2019) Polity 2 and Current Affairs (November 2018)

The Bill amends the Citizenship Act, 1955 to make illegal migrants who are Hindus, Sikhs, Buddhists, Jains, Parsis and Christians from Afghanistan, Bangladesh and Pakistan, eligible for citizenship.

Under the Act, one of the requirements for citizenship by naturalisation is that the applicant must have resided in India during the last 12 months, and for 11 of the previous 14 years. The Bill relaxes this 11 year requirement to six years for persons belonging to the same Six religions and three countries.

The Bill provides that the registration of Overseas Citizen of India (OCI) cardholders may be cancelled if they violate any law

Hence, option(c) is the correct answer. 78. Articles 301 to 307 in Part XIII of the Indian Constitution deal with the trade,

commerce and intercourse within the territory of India. In this context, consider the following statements: (1) The Parliament of India cannot give preference to one State over another or

discriminate between the States except in the case of scarcity of goods in any part of India.

(2) The State legislature can give preference to one State over another in public interest and economic viability.

Which of the statements given above is/are correct? (a) 1 only (b) 2 only (c) Both 1 and 2 (d) Neither 1 nor 2

Answer: (a) Explanation: The freedom guaranteed by Article 301 is a freedom from all restrictions, except those which are provided for in the other provisions (Articles 302 to 305) of Part XIII of the Constitution itself. These are explained below: 1. The Indian Parliament can impose restrictions on the freedom of trade, commerce and

intercourse between the States or within a State in public interest, but cannot give preference to one State over another or discriminate between the States except in the case of scarcity of goods in any part of India. Hence, Statement 1 is correct.

2. The legislature of a State can impose reasonable restrictions on the freedom of trade, commerce and intercourse with that State or within that State in public interest. But, a Bill for this purpose can be introduced in the legislature only with the previous sanction of the President. Further, the State legislature cannot give preference to one State over another or discriminate between the States. Hence, Statement 2 is not correct.

Additional Information:

The legislature of a State can impose on goods imported from other States or the Union territories any tax to which similar goods manufactured in that State are subject. This provision prohibits the imposition of discriminatory taxes by the State.

The freedom (under Article 301) is subject to the nationalisation laws (i.e., laws providing for monopolies in favour of the Centre or the States).

Source: M Laxmikanth: Indian Polity: Ch. 15: Inter-State relations, Page No. 15.3

Page 54: Answer Key & Exp for GS Prelim Test-8 held on 20 …...1 Polity 2 Prelim IAS Test Series (2019) –GS Test 8 (20.01.2019) and Current Affairs (November 2018) Answer Key & Exp for GS

54

Prelim IAS Test Series (2019) – GS Test 8 (20.01.2019) Polity 2 and Current Affairs (November 2018)

79. Model Code of Conduct is a set of guidelines issued by the Election Commission of

India for conduct of political parties and candidates during elections. Which of the following are permissible activities under the Model Code of Conduct? (1) On-going well-being programmes, which actually started before the

announcement of elections. (2) Criticism of other political parties and candidates related to their policies,

programmes, past record and work. (3) Aspect of the private life, not connected with the public activities, of the

leaders or workers of other parties shall be permitted to be criticized. (4) Issue of advertisement at the cost of public exchequer in the newspapers and

other media. Select the correct answer using the code given below: (a) 1 and 2 only (b) 1, 2 and 3 only (c) 2, 3 and 4 only (d) 1, 2, 3 and 4

Answer: (a) Explanation: The MCC is a set of guidelines issued by the Election Commission to regulate political parties and candidates prior to elections, to ensure free and fair elections. This is in keeping with Article 324 of the Constitution, which gives the Election Commission the power to supervise elections to the Parliament and State Legislatures. The MCC is operational from the date that the election schedule is announced till the date that results are announced. Statement 1 is correct. The party in power whether at the Centre or in the State or States concerned shall ensure that no cause is given for any complaint that it has used its official position for the purposes of its election campaign. Ministers and other authorities shall not sanction grants/payments out of discretionary funds from the time elections are announced by the Commission. However, there is no bar on On-going programmes, which actually started before the announcement of elections. Statement 2 is correct. Criticism of other political parties, when made, shall be confined to their policies and programme, past record and work. Statement 3 is not correct. Parties and Candidates shall refrain from criticism of all aspects of private life, not connected with the public activities of the leaders or workers of other parties. Criticism of other parties or their workers based on unverified allegations or distortion shall be avoided. Statement 4 is not correct. Issue of advertisement at the cost of public exchequer in the newspapers and other media and the misuse of official mass media during the election period for partisan coverage of political news and publicity regarding achievements with a view to furthering the prospects of the party in power shall be scrupulously avoided. Sources:

https://eci.gov.in/mcc/

https://www.eci.nic.in/archive/handbook/candidates/cap10/cap10_3.htm

Page 55: Answer Key & Exp for GS Prelim Test-8 held on 20 …...1 Polity 2 Prelim IAS Test Series (2019) –GS Test 8 (20.01.2019) and Current Affairs (November 2018) Answer Key & Exp for GS

55

Prelim IAS Test Series (2019) – GS Test 8 (20.01.2019) Polity 2 and Current Affairs (November 2018)

80. Which of the following rights is/are ensured under the Right to freedom of religion in

the Indian Constitution only to the Minorities? (1) Right to establish educational institutions (2) Right to establish charitable institutions (3) Right to establish religious institutions Select the correct answer using the code given below: (a) 1 only (b) 3 only (c) 2 and 3 only (d) None of the above

Answer: (d) Explanation: Firstly, no provision of the Right to Freedom of Religion (under Articles 25-28) is specific to the Minorities. Hence, option (d) is the correct answer. Statement 1 is incorrect. Further, Right to establish educational institutions is not a right under Right to Freedom of Religion but under Article 30, Cultural and Educational Rights. Article 30(1): All minorities, whether based on religion or language, shall have the right to establish and administer educational institutions of their choice. Statements 2 and 3 are incorrect. Article 26: Right to establish institutions for charitable and religious purposes under Right to Freedom of Religion is available to any section of any religious denomination and not only to Minorities. Article 26: Subject to public order, morality and health, every religious denomination or any section thereof shall have the right—

To establish and maintain institutions for religious and charitable purposes;

To manage its own affairs in matters of religion; Prohibition of employment of children in factories, etc. Freedom of conscience and free profession, practice and propagation of religion. Prohibition of traffic in human beings and forced labour. Freedom to manage religious affairs.

To own and acquire movable and immovable property; and to administer such property in accordance with law.

Source: Indian Polity by M. Laxmikanth, Chapter 7, Fundamental Rights; Page No. 7.16-7.17 81. With reference to ‘STAPCOR’, sometimes seen in the news, consider the following

statements: (1) It aims to have an international conference every year to review the status and

progress of coral reefs all over the world. (2) STAPCOR – 2018 was held at Bangaram coral Island of Lakshadweep under

the theme ‘Reef for Life’. Which of the statements given above is/are correct? (a) 1 only (b) 2 only (c) Both 1 and 2

Page 56: Answer Key & Exp for GS Prelim Test-8 held on 20 …...1 Polity 2 Prelim IAS Test Series (2019) –GS Test 8 (20.01.2019) and Current Affairs (November 2018) Answer Key & Exp for GS

56

Prelim IAS Test Series (2019) – GS Test 8 (20.01.2019) Polity 2 and Current Affairs (November 2018)

(d) Neither 1 nor 2 Answer: (b) Explanation: Statement 1 is incorrect. The aim of STAPCOR is to have an international conference in every 10 years to review the status and progress of coral reefs all over the world. The International Conference on Status and Protection of Coral Reefs (STAPCOR – 2018) was held at Bangaram coral Island of Lakshadweep. Statement 2 is correct. The International Conference on Status and Protection of Coral Reefs (STAPCOR – 2018) with the theme “Reef for Life” was inaugurated by the Union Minister of Environment, Forest and Climate Change, at Bangaram coral Island of Union Territory of Lakshadweep. 82. In the context of 'Green Climate Fund (GCF)', consider the following statements:

(1) The general concept for GCF was first proposed at the Conference of Parties (CoP) to the UNFCCC in Copenhagen, Denmark.

(2) The COP in Paris, France decided to establish GCF. (3) GCF aims to deliver equal amounts of funding for mitigation and adaptation. (4) The fund is created to support the efforts of all the member countries of the

UN to respond to the challenge of climate change. Select the correct answer using the code given below: (a) 1 and 3 only (b) 2 and 4 only (c) 1, 2 and 3 only (d) 1, 2, 3 and 4

Answer: (a) Explanation: Statement 1 is correct. The general concept for GCF was first proposed at the Conference of the Parties (COP) to the UNFCCC in Copenhagen, Denmark (COP 15) in 2009. Statement 2 is incorrect. In 2010 the COP in Cancun, Mexico (COP 16), decided to establish the GCF. Statement 3 is correct. GCF aims to deliver equal funding for mitigation and adaptation on need basis. Statement 4 is incorrect. The Green Climate Fund (GCF) is a new global fund created to support the efforts of developing countries to respond to the challenge of climate change. GCF helps developing countries limit or reduce their green house gas (GHG) emissions and adapt to climate change. GCF was set up by 194 countries which are parties to the UNFCCC in 2010. Additional Information:

Recently GCF has approved US $43.4 million for enhancing climate resilience for millions of people living in India’s coastal communities as part of its efforts to combat extreme impacts of climate change.

Page 57: Answer Key & Exp for GS Prelim Test-8 held on 20 …...1 Polity 2 Prelim IAS Test Series (2019) –GS Test 8 (20.01.2019) and Current Affairs (November 2018) Answer Key & Exp for GS

57

Prelim IAS Test Series (2019) – GS Test 8 (20.01.2019) Polity 2 and Current Affairs (November 2018)

83. In the context of the ‘Pradhan Mantri Fasal Bima Yojana (PMFBY)’, which of the following statements is /are correct? (1) Under this scheme, farmers need to pay a uniform premium of only 2% for all

kharif, rabi and horticultural crops. (2) It replaced two schemes - National Agricultural Insurance Scheme and

Modified National Agricultural Insurance Scheme. (3) One of the objectives of PMFBY is to encourage farmers to adopt innovative

and modern agricultural practices. (4) It does not provide insurance benefits to landless labourers. Select the correct answer using the code given below: (a) 1 and 3 only (b) 2 and 3 only (c) 1, 2 and 3 only (d) 1, 2, 3 and 4

Answer: (b) Explanation: Statement 1 is not correct. Under this scheme, farmers need to pay uniform premium of only 2% for all Kharif crops and 1.5% for all Rabi crops. In case of annual commercial and horticultural crops, farmers have to pay premium of only 5%. It includes all farmers growing notified crops in notified area during season who have insurable interest in crop are eligible. It also provides insurance benefits to landless labourers. It is also compulsory for loanee farmers availing crop loans for notified crops in notified areas and voluntary for non-loanee farmers. Statement 2 is correct. It replaced two schemes - National Agricultural Insurance Scheme and Modified National Agricultural Insurance Scheme. Statement 3 is correct. One of the objectives of PMFBY is to encourage farmers to adopt innovative and modern agricultural practices. Statement 4 is not correct. It does provide insurance benefits to landless labourers. 84. With reference to the United Nations Human Rights Council (UNHRC), consider the

following statements: (1) The five permanent members of the United Nations Security Council are also

the members of the UNHRC. (2) Recently, India was elected for the fifth time to UNHRC by the UN General

Assembly. Which of the statements given above is/are correct? (a) 1 only (b) 2 only (c) Both 1 and 2 (d) Neither 1 nor 2

Answer: (b) Explanation:

Page 58: Answer Key & Exp for GS Prelim Test-8 held on 20 …...1 Polity 2 Prelim IAS Test Series (2019) –GS Test 8 (20.01.2019) and Current Affairs (November 2018) Answer Key & Exp for GS

58

Prelim IAS Test Series (2019) – GS Test 8 (20.01.2019) Polity 2 and Current Affairs (November 2018)

The United Nations Human Rights Council (UNHRC) is a United Nations body whose mission is to promote and protect human rights around the world. Statement 1 is not correct. The UNHRC has 47 members elected for staggered three-year terms on a regional group basis. The United States under President Donald Trump's Administration had withdrawn from UNHRC in 2018 after it questioned the legitimacy of the Council because of presence of several dictatorial regimes violating human rights on it. Statement 2 is correct. India was elected to the United Nations' top human rights body UNHRC in 2018 for a period of three years beginning January 1, 2019, getting 188 votes in the Asia-Pacific category, the highest number of votes among all candidates. 85. In the context of which of the following do you sometimes find the terms 'SWAS',

'SAFAL', and 'STAR', in the news? (a) These are less polluting green firecrackers developed by CSIR. (b) A series of UAV developed by Amazon to transport packages, food or other

goods. (c) Various names given to the electric cars developed in India by Mahindra

Electric. (d) These are the three variants of India's first unmanned tank developed by

DRDO. Answer: (a) Explanation: CSIR scientists have developed Less Polluting Firecrackers which are not only environment friendly but 15-20 % cheaper than the conventional ones. These crackers have been named as Safe Water Releaser (SWAS), Safe Minimal Aluminium (SAFAL) and Safe Thermite Cracker (STAR). It has the unique property of releasing water vapour and /or air as dust suppressant and dilutent for gaseous emissions and matching performance in sound with conventional crackers. Hence, option (a) is the correct answer. 86. With reference to ‘the Cape Town Convention/Protocol’, sometimes seen in the

news, consider the following statements: (1) It seeks to achieve efficient financing of high value mobile equipments, like

airframes, helicopters and engines, in order to make the operations cost effective and affordable.

(2) The Cape Town Convention is related to aviation sector only. (3) India became a party to the Convention/Protocol in 2018. Which of the statements given above is/are not correct? (a) 1 and 2 only (b) 1 and 3 only (c) 2 and 3 only (d) 1, 2 and 3

Answer: (c)

Page 59: Answer Key & Exp for GS Prelim Test-8 held on 20 …...1 Polity 2 Prelim IAS Test Series (2019) –GS Test 8 (20.01.2019) and Current Affairs (November 2018) Answer Key & Exp for GS

59

Prelim IAS Test Series (2019) – GS Test 8 (20.01.2019) Polity 2 and Current Affairs (November 2018)

Explanation: The Ministry of Civil Aviation released the draft Cape Town Convention Bill, 2018. The draft Bill seeks to implement the Cape Town Convention (Convention on International Interests in Mobile Equipment), and Protocol (Protocol to the Convention on Matters Specific to Aircraft Equipment) in India. The Cape Town Convention/Protocol was adopted in Cape Town in November, 2001. Statement 1 is correct. The Convention/Protocol primarily seeks to achieve efficient financing of high value mobile equipment, like airframes, helicopters and engines, in order to make the operations cost effective and affordable. Statement 2 is incorrect. While Cape Town Convention is for three sectors - aviation, railways and space equipment, there are separate protocols for each of the three. The aircraft protocol was adopted at Cape Town in 2001. Statement 3 is incorrect. India became a party to the Convention/Protocol in July, 2008. 87. Which of the following is a purpose of 'SPARC’, a scheme of the Government of

India? (a) A flagship programme aimed at all round development of school children. (b) It is for helping unemployed youth to secure bank loans. (c) To improve the research ecosystem of India’s higher educational institutions. (d) To create and provide sustainable self-employment opportunities to one

million educated youths. Answer: (c) Explanation: The Ministry of Human Resource Development has launched this scheme. The Scheme for Promotion of Academic and Research Collaboration (SPARC) aims at improving the research ecosystem of India’s Higher Educational Institutions by facilitating academic and research collaborations between Indian Institutions and the best institutions in the world from 28 selected nations to jointly solve problems of national and international relevance. Hence, option (c) is the correct answer. 88. With reference to the Defence Acquisition Council, which of the following statements

is/are not correct? (1) It is headed by the Prime Minister of India. (2) It clears all defence acquisitions including imported as well as those produced

indigenously. (3) It takes the decision regarding ‘offset’ provisions in respect of acquisition

proposals above Rs. 300 crores. Select the correct answer using the code given below: (a) 1 only (b) 2 and 3 only (c) 1 and 3 only (d) 1, 2 and 3

Page 60: Answer Key & Exp for GS Prelim Test-8 held on 20 …...1 Polity 2 Prelim IAS Test Series (2019) –GS Test 8 (20.01.2019) and Current Affairs (November 2018) Answer Key & Exp for GS

60

Prelim IAS Test Series (2019) – GS Test 8 (20.01.2019) Polity 2 and Current Affairs (November 2018)

Answer: (a) Explanation: To counter corruption and speed up decision-making in military procurement, the government of India in 2001 decided to set up an integrated Defence Acquisition Council (DAC). Statement 1 is not correct: The Defence Acquisition Council (DAC) is headed by the Defence Minister. Statement 2 is correct: The DAC is responsible to give policy guidelines on acquisitions, based on long-term procurement. It also clears all acquisitions, which includes both imported and those produced indigenously or under a foreign license. Statement 3 is correct: The DAC take the decision regarding ‘offset’ provisions in respect of acquisition proposals above Rs. 300 crores. 89. Recently the Geographical Indication (GI) tag has been granted to Alphonso mango.

In this context, consider the following statements: (1) GI tag is primarily given only to agricultural and natural products. (2) The registration of GI tag is valid only for 10 years. (3) The first GI tag in India was given to the Nagpur orange. (4) At international level GI is governed by Trade-Related Aspects of Intellectual

Property Rights (TRIPS). Select the correct answer using the code given below: (a) 1 and 2 only (b) 2 and 4 only (c) 2, 3 and 4 only (d) 1, 2, 3 and 4

Answer: (b) Explanation: Alphonso from Ratnagiri, Sindhudurg, Palghar, Thane and Raigad districts of Maharashtra, is registered as Geographical Indication (GI). A Geographical Indication or a GI is an indication used on products that have a specific geographical origin and possess qualities or a reputation that are due to that origin. There are a total of 325 products from India that carry this indication. Statement 1 is not correct. It is primarily given to agricultural, natural and manufactured goods having special quality and established reputation. Statement 2 is correct. The registration of GI is valid for 10 years after which it needs to be renewed. Statement 3 is not correct. The first product to get a GI tag in India was Darjeeling tea in 2004. Statement 4 is correct. At international level GI is governed by Trade-Related Aspects of Intellectual Property Rights (TRIPS). Violation of GI tags is punishable offence under law. 90. In the context of 'Global Competitiveness Report - 2018', which of the following

statements is/are correct?

Page 61: Answer Key & Exp for GS Prelim Test-8 held on 20 …...1 Polity 2 Prelim IAS Test Series (2019) –GS Test 8 (20.01.2019) and Current Affairs (November 2018) Answer Key & Exp for GS

61

Prelim IAS Test Series (2019) – GS Test 8 (20.01.2019) Polity 2 and Current Affairs (November 2018)

(1) India has improved its ranking by five places and stood first among BRICS countries.

(2) The Global Competitiveness Report is a yearly report published by the World Bank.

Select the correct answer using the code given below: (a) 1 only (b) 2 only (c) Both 1 and 2 (d) Neither 1 nor 2

Answer: (d) Explanation: Statement 1 is not correct. India has improved its ranking by five places; the largest gain among all G-20 countries; to emerge as the world’s 58th most competitive economy in the WEF’s latest Global Competitiveness Index. BRICS Ranking – Brazil (72), Russia (43), India (58), China (28), South Africa (67). Statement 2 is not correct. The Global Competitiveness Report (GCR) is a yearly report published by the World Economic Forum. Since 2004, the Global Competitiveness Report ranks countries based on the Global Competitiveness Index. 91. Countries of the Indian Ocean Rim Association (IORA) have recently adopted the

Delhi Declaration. With which of the following matters is it related to? (a) Global Warming (b) Maritime Security (c) Renewable Energy (d) None of the above

Answer: (c) Explanation: 21 countries in the Indian Ocean Rim Association (IORA) recently adopted the Delhi Declaration on Renewable Energy in the Indian Ocean Region. The declaration was the outcome of the 2nd IORA Renewable Energy Ministerial Meeting. The Indian Ocean Rim Association was set up with the objective of strengthening regional cooperation and sustainable development within the Indian Ocean Region with 21 Member States and 7 Dialogue Partners. Hence, option (c) is the correct answer. 92. ‘SATAT’ an initiative of the Government of India, aims to:

(a) Promote compressed Bio-Gas production (b) Provide satellite internet to people who still don’t have reliable access (c) Promote sustainable lifestyle through use of recycled products (d) Provide an up-to-date picture of the state of land use and land cover on a

large scale

Page 62: Answer Key & Exp for GS Prelim Test-8 held on 20 …...1 Polity 2 Prelim IAS Test Series (2019) –GS Test 8 (20.01.2019) and Current Affairs (November 2018) Answer Key & Exp for GS

62

Prelim IAS Test Series (2019) – GS Test 8 (20.01.2019) Polity 2 and Current Affairs (November 2018)

Answer: (a) Explanation: The Ministry of Petroleum and Natural Gas has launched SATAT initiative to promote Compressed Bio-Gas (CBG) as an alternative, green transport fuel. Titled SATAT, the initiative is aimed at providing a Sustainable Alternative Towards Affordable Transportation (SATAT) as a developmental effort that would benefit vehicle-users as well as farmers and entrepreneurs. Hence, option (a) is the correct answer. 93. With reference to ‘the Financial Action Task Force (FATF)’, sometimes seen in the

news, consider the following statements: (1) It is an inter-governmental body established in 2008 on the initiative of the G-

7. (2) The FATF Secretariat is housed at the (Organisation for Economic Co-

operation and Development) OECD headquarters in Paris. (3) The main objective of the FATF is to improve the banking sector's ability to

absorb shocks arising from financial and economic stress. Which of the statements given above is/are correct? (a) 3 only (b) 1 and 2 only (c) 2 and 3 only (d) 1, 2 and 3

Answer: (b) Explanation: Statement 1 is correct. The Financial Action Task Force (FATF) is an inter-governmental body established in 1989 on the initiative of the G7. Statement 2 is correct. The FATF Secretariat is housed at the OECD headquarters in Paris. Statement 3 is not correct. The objectives of the FATF are to set standards and promote effective implementation of legal, regulatory and operational measures for combating money laundering, terrorist financing and other related threats to the integrity of the international financial system. 94. Recently the Union Ministry of Environment, Forests and Climate Change

(MoEFCC) has reconstituted the Environment Pollution (Prevention and Control) Authority (EPCA). In this context, consider the following statements: (1) EPCA was constituted with the objective of protecting and improving the

quality of the environment and controlling environmental pollution in all the major cities of India.

(2) EPCA is a Supreme Court mandated body formed under the Environment Protection Act, 1986.

Which of the statements given above is/are correct? (a) 1 only (b) 2 only

Page 63: Answer Key & Exp for GS Prelim Test-8 held on 20 …...1 Polity 2 Prelim IAS Test Series (2019) –GS Test 8 (20.01.2019) and Current Affairs (November 2018) Answer Key & Exp for GS

63

Prelim IAS Test Series (2019) – GS Test 8 (20.01.2019) Polity 2 and Current Affairs (November 2018)

(c) Both 1 and 2 (d) Neither 1 nor 2

Answer: (b) Explanation: Statement 1 is not correct. Environment Pollution Control Authority (EPCA) was constituted with the objective of ‘protecting and improving’ the quality of the environment and ‘controlling environmental pollution’ in the National Capital Region. The EPCA also assists the apex court in various environment-related matters in the region. Statement 2 is correct. EPCA is a Supreme Court mandated body tasked with taking various measures to tackle air pollution in the National Capital Region. It was notified in 1998 by Environment Ministry under Environment Protection Act, 1986. 95. With reference to ‘the International Social Security Association (ISSA)’, sometimes

seen in the news, consider the following statements: (1) ISSA was founded in 2007 under the auspices of the United Nations

Commission on Human Rights. (2) It is an association of national social security administrations of all the

developing countries. (3) Recently, the Ministry of Social Justice and Empowerment has won the ‘ISSA

Good Practice Award’ for administrative solution for coverage extension. Which of the statements given above is/are not correct? (a) 2 only (b) 1 and 3 only (c) 2 and 3 only (d) 1, 2 and 3

Answer: (d) Explanation: Statement 1 is not correct. The International Social Security Association (ISSA) is the principal international organization for Social Security Organizations, Governments and Departments of Social Security. The ISSA was founded in 1927 under the auspices of the International Labour Organization (ILO), Geneva. Statement 2 is not correct. It promotes excellence in social security administration through professional guidelines, expert knowledge, services and support to enable its Members to develop dynamic social security systems (It’s not an association of all the developing countries). Statement 3 is not correct. The Employees’ State Insurance Corporation (ESIC) has won the ‘ISSA Good Practice Award’ for Administrative Solution for Coverage Extension at the “Regional Social Security Forum for Asia and the Pacific” held at Kuala Lumpur, Malaysia recently. The award recognizes the measures taken by ESIC for extension of coverage - SPREE (Scheme for Promoting Registration of Employers and Employees), reduced rate of contribution rates for 24 months in newly implemented areas and raising the wage limit for coverage under the ESI Act, etc.

Page 64: Answer Key & Exp for GS Prelim Test-8 held on 20 …...1 Polity 2 Prelim IAS Test Series (2019) –GS Test 8 (20.01.2019) and Current Affairs (November 2018) Answer Key & Exp for GS

64

Prelim IAS Test Series (2019) – GS Test 8 (20.01.2019) Polity 2 and Current Affairs (November 2018)

96. The Gujarat Forest Department has recently vaccinated the lions in Gir sanctuary to protect them from a deadly Canine Distemper Virus (CDV). In this context, consider the following statements: (1) CDV is a viral disease that infects the gastrointestinal, respiratory, and central

nervous systems. (2) Dogs who have not been vaccinated for Canine Distemper are the most at-

risk. (3) CDV can only be spread through direct contact. (4) Inhaling the virus is the primary method of exposure. Select the correct answer using the code given below: (a) 1, 2 and 4 only (b) 2, 3 and 4 only (c) 1, 2 and 3 only (d) 1, 2, 3 and 4

Answer: (a) Explanation: Statement 1 is correct. Canine Distemper Virus (CDV) is a viral disease that infects the gastrointestinal, respiratory, and central nervous systems. Statement 2 is correct. Dogs who have not been vaccinated for Canine Distemper are the most at-risk. While the disease can also be contracted when improperly vaccinated or when a dog has high susceptibility to bacterial infection, these cases are rare. Statement 3 is not correct. CDV can be spread through direct contact (licking, breathing air, etc.) or indirect contact (bedding, toys, food bowls, etc.), though it cannot live on surfaces for very long. Statement 4 is correct. Inhaling the virus is the primary method of exposure. There is no known cure for CDV. 97. With reference to the 'Rashtriya Vayoshri Yojana', which of the following statements

is/ are correct? (1) The scheme is for providing Physical Aids and Assisted-living Devices for all

senior citizens belonging to all communities. (2) It is a Centrally Sponsored Scheme where Centre will bear 60% of the cost,

and 40% will be borne by the States. (3) The expenditure for implementation of the scheme will be met from the senior

citizens welfare fund. Select the correct answer using the code given below: (a) 3 only (b) 1 and 3 only (c) 2 and 3 only (d) 1, 2 and 3

Answer: (a) Explanation:

Page 65: Answer Key & Exp for GS Prelim Test-8 held on 20 …...1 Polity 2 Prelim IAS Test Series (2019) –GS Test 8 (20.01.2019) and Current Affairs (November 2018) Answer Key & Exp for GS

65

Prelim IAS Test Series (2019) – GS Test 8 (20.01.2019) Polity 2 and Current Affairs (November 2018)

Statement 1 is incorrect. Rashtriya Vayoshri Yojana scheme aims at providing Senior Citizens, belonging to BPL category and suffering from any of the age related disability/infirmity Low vision, Hearing impairment, Loss of teeth and Locomotor disability, with such assisted-living devices which can restore near normalcy in their bodily functions, overcoming the disability/infirmity manifested. Statement 2 is incorrect. This is a Central Sector Scheme, fully funded by the Central Government. Statement 3 is correct. The expenditure for implementation of the scheme will be met from the Senior Citizens’ Welfare Fund. Under the scheme, free of cost distribution of the devices, commensurate with the extent of disability/infirmity that is manifested among the eligible senior citizens will take place. 98. With reference to ‘the Champions of the Earth Award’, sometimes seen in the news,

consider the following statements: (1) It is the highest environmental award given by the Global Environment Facility. (2) The award is given exclusively to the Head of the States whose actions have

had a transformative positive impact on the environment. (3) The Prime Minister of India has been given this award recently for his

leadership of the ‘International Solar Alliance’ and ‘pledge to eliminate single use plastic in India by 2022’.

Which of the statements given above is/are correct? (a) 3 only (b) 1 and 2 only (c) 2 and 3 only (d) 1, 2 and 3

Answer: (a) Explanation: Statement 1 is not correct. ‘Champions of the Earth’, the UN’s highest environmental honour, celebrates outstanding figures from the public and private sectors and from civil society whose actions have had a transformative positive impact on the environment. Statement 2 is not correct. Launched in 2005, Champions of the Earth has recognized dozens of exemplary individuals and organizations. Through their extraordinary achievements – whether through political leadership, grassroots action, scientific innovation, or entrepreneurial vision – each of these Champions has inspired critical action on behalf of the global environment. Statement 3 is correct. The Prime Minister of India has been awarded this award recently for his leadership of the ‘International Solar Alliance’ and ‘pledge to eliminate single use plastic in India by 2022’. 99. In the context of the 'South-East Asia Regulatory Network (SEARN)', consider the

following statements: (1) It promotes regulatory and health collaboration among the countries of South-

East Asia Region. (2) SEARN includes all the ASEAN countries.

Page 66: Answer Key & Exp for GS Prelim Test-8 held on 20 …...1 Polity 2 Prelim IAS Test Series (2019) –GS Test 8 (20.01.2019) and Current Affairs (November 2018) Answer Key & Exp for GS

66

Prelim IAS Test Series (2019) – GS Test 8 (20.01.2019) Polity 2 and Current Affairs (November 2018)

(3) It is based at the London School of Hygiene & Tropical Medicine. (4) SEARN is developed by the Centre for Development of Advanced Computing. Select the correct answer using the code given below: (a) 2 and 3 only (b) 2 and 4 only (c) 1, 2 and 3 only (d) 1, 2, 3 and 4

Answer: (d) Explanation: Statements 1 and 2 are correct. Information Sharing Platform Gateway for South-East Asia Regulatory Network (SEARN) developed by Centre for Development of Advanced Computing has been launched recently. It will promote regulatory and health collaboration among the countries of the South-East Asia Region. Statements 3 and 4 are correct. SEARN includes all ASEAN countries. In the South-East Asia region in WHO, India is actively contributing & providing support for the SEARN to guarantee access to high-quality medical products. SEARN is based at the London School of Hygiene & Tropical Medicine, is a platform to facilitate research collaboration. 100. With reference to the 'Global Soil Bio-diversity Atlas', consider the following

statements: (1) The Global Soil Biodiversity Atlas is prepared by the World Wide Fund for

Nature. (2) India is among the nations that face moderate level of danger to soil

biodiversity. (3) Soil biodiversity encompasses the presence of micro-organisms, micro-fauna

and macro-fauna. Select the correct answer using the code given below: (a) 1 and 2 only (b) 2 and 3 only (c) 1 and 3 only (d) 1, 2 and 3

Answer: (c) Explanation: Statement 1 is correct. India’s soil biodiversity is in grave peril, according to the Global Soil Biodiversity Atlas prepared by the World Wide Fund for Nature. Statement 2 is not correct. The WWF’s ‘risk index’ for the globe - indicating threats from loss of above-ground diversity, pollution and nutrient over-loading, over-grazing, intensive agriculture, fire, soil erosion, desertification and climate change - shows India among countries whose soil biodiversity faces the highest level of risk. Statement 3 is correct. Soil biodiversity encompasses the presence of micro-organisms, micro-fauna (nematodes and tardigrades for example), and macro-fauna (ants, termites and earthworms). The findings were part of the bi-annual Living Planet Report 2018.

Page 67: Answer Key & Exp for GS Prelim Test-8 held on 20 …...1 Polity 2 Prelim IAS Test Series (2019) –GS Test 8 (20.01.2019) and Current Affairs (November 2018) Answer Key & Exp for GS

67

Prelim IAS Test Series (2019) – GS Test 8 (20.01.2019) Polity 2 and Current Affairs (November 2018)